You are on page 1of 35

https://t.me/IAS201819 https://t.me/PDF4Exams https://t.

me/PDF4Exams

www.nextias.com

Saket Centre : 316/274, Westend Marg, Saidulajab, Near Saket Metro Station, Delhi-30
Old Rajinder Nagar Centre : Ground Floor, 6, Old Rajinder Nagar (Near Salwan School Gate No.-2) New Delhi-60
Ph: 011-45124642, 8800338066  |  E-mail : info@nextias.com

CSE (Prelims) Test Series-2019


General Studies  |  Test-4

A
Booklet
Time Allowed : 2 hrs.
Maximum Marks: 200
Test Code: 190904

INSTRUCTIONS
Please read each of the following instructions carefully before attempting questions:
1. IMMEDIATELY AFTER THE COMMENCEMENT OF THE EXAMINATION, YOU SHOULD CHECK THAT THIS
TEST BOOKLET DOES NOT HAVE ANY UNPRINTED OR TORN OR MISSING PAGES OR ITEMS, ETC. IF SO,
GET IT REPLACED BY A COMPLETE TEST BOOKLET.
2. Please note that it is the candidate’s responsibility to encode and fill in the Roll Number and Test Booklet
Series A, B, C or D carefully and without any omission or discrepancy at the appropriate places in the OMR
Answer Sheet. Any omission/discrepancy will render the Answer Sheet liable for rejection.
3. You have to enter your Roll Number on the OMR Answer Sheet in the box provided.
4. This Test Booklet contains 100 items (Questions). Each item is printed in English only. Each item comprises
four responses (Answers). You will select the response which you want to mark on the Answer Sheet. In case
you feel that there is more than one correct response, mark the response which you consider the best. In any
case, choose ONLY ONE response for each item.
5. You have to mark all your responses ONLY on the separate answer sheet provided. See directions in the
Answer Sheet.
6. All items carry equal marks.
7. Before you proceed to mark in the Answer Sheet the response to various items in the Test Booklet, you have
to fill in some particulars in the Answer Sheet as per instructions sent to you with your Admission Certificate.
8. After you have completed filling in all your responses on the Answer Sheet and the examination has concluded,
you should hand over to the invigilator only the Answer Sheet. You are permitted to take away with you the Test
Booklet.
9. Sheets for rough work are appended in the Test Booklet at the end.
10. Penalty for wrong answers:
THERE WILL BE PENALTY FOR WRONG ANSWERS MARKED BY A CANDIDATE IN THE OBJECTIVE TYPE
QUESTION PAPERS.
(i) There are four alternatives for the answer to every question. For each question for which a wrong answer
has been given by the candidate, one-third of the marks assigned to that question will be deducted as
penalty.
(ii) If a candidate gives more than one answer, it will be treated as a wrong answer even if one of the given
answers happens to be correct and there will be same penalty as above to that question.
(iii) If question is left blank, i.e., no answer is given by the candidate, there will be no penalty for that question.
11. CHALLENGE THE QUESTION: If students feel that either the question(s)/answer(s) needs to be modified or
require clarification, they can email at feedback@nextias.com by 5 pm, 22.10.2018.

DO NOT OPEN THIS BOOKLET UNTIL YOU ARE ASKED TO DO SO


For any queries kindly email at: feedback@nextias.com
https://t.me/TheHindu_Zone_official
https://t.me/IAS201819 https://t.me/PDF4Exams https://t.me/PDF4Exams

2 | CSE (Prelims) Test Series-2019

Objective Questions

Q.1 With reference to support prices provided to (c) United Nation’s Population Division
farmers by the Government of India, consider (d) Asian Development Bank
the following statements:
1. Minimum Support Price (MSP) is a price Q.6 With reference to share of various items in
below which purchase of certain farm India’s recent external trade (in terms of
produce from farmers is statutorily value), consider the following statements:
prohibited. 1. India exports more electronic goods than
2. Minimum Support Price (MSP) is engineering goods.
announced by the Government every 2. India imports more gems and jewellery
year in annual budget for certain specified than petroleum oil.
crops
3. Commission for Agricultural Costs and Which of the statements given above is/are
Prices (CACP) recommends Fair and correct?
Remunerative Price(FRP) for sugarcane. (a) 1 only (b) 2 only
(c) Both 1 and 2 (d) Neither 1 nor 2
Which of the statements given above is/are
not correct? Q.7 The term “Aggregate Measurement of
(a) 1 only (b) 2 and 3 only Support (AMS)” is popularly associated with
(c) 1 and 2 only (d) 1, 2 and 3 which organisation/association?
Q.2 With reference to labour force participation (a) ASEAN
rate (LFPR) in India, consider the following (b) United Nations Development Programme
statements: (c) WTO
1. It is calculated by expressing the number (d) World Bank
of persons employed as a percentage of
the working-age population. Q.8 With reference to the global benchmark tool
2. As per Fourth Annual Employment- “Logistics Performance Index (LPI)”, consider
Unemployment Survey by the Labour the following statements:
Bureau, LFPR for the urban female is 1. It is an annual index released by United
significantly higher than rural female. Nations Conference on Trade and
Which of the statements given above is/are Development (UNCTAD).
correct? 2. India’s ranking in the index has steadily
(a) 1 only (b) 2 only improved over the years.
(c) Both 1 and 2 (d) Neither 1 nor 2 Which of the statements given above is/are
Q.3 “World Population Prospectus” is released correct?
by which of the following organisation? (a) 1 only (b) 2 only
(a) United Nations Development Programme. (c) Both 1 and 2 (d) Neither 1 nor 2
(b) United Nations.
(c) World Bank. Q.9 With reference to Pradhan Mantri Gram Sadak
(d) World Economic Forum. Yojana (PMGSY), consider the following
statements:
Q.4 Which of the following can be considered as
1. To provide single connectivity by way
consequences of negative trade balance?
of an All-Weather road to unconnected
1. Widening of current account deficit of a
country. habitations.
2. Depreciation of National currency. 2. PMGSY has built in provisions for road
safety under the scheme.
Select the correct answer using the code given 3. A system for use of geo-spatial technology
below
for mapping rural roads built under
(a) 1 only (b) 2 only
PMGSY has been developed.
(c) Both 1 and 2 (d) Neither 1 nor 2
Q.5 Which of the following organisation releases Which of the statements given above is/are
the “Report on Migration and Remittances”? correct?
(a) World Bank (a) 1 only (b) 1 and 3 only
(b) International Monetary Fund (c) 2 and 3 only (d) 1, 2 and 3

© Copyright:   www.nextias.com

https://t.me/TheHindu_Zone_official
https://t.me/IAS201819 https://t.me/PDF4Exams https://t.me/PDF4Exams

Economy, Revision of Test-3 & Current Affairs_190904 | 3

Q.10 With reference to Rashtriya Uchchatar Shiksha Q.14 A new MPI has been released in 2018 with
Abhiyan (RUSA), consider the following several modifications. With reference to newly
statements: released global Multidimensional Poverty
1. It was launched to provide funding to the Index (MPI), consider the following statements:
1. It assesses poverty at the individual level.
Universities with ‘Institute of Eminence’ 2. It was originally developed by World Bank
status. with the United Nations Development
2. National Mission Authority under RUSA is Programme (UNDP) in 2010.
headed by the Prime Minister.
Which of the statements given above is/are
Which of the statements given above is/are correct?
correct? (a) 1 only (b) 2 only
(c) Both 1 and 2 (d) Neither 1 nor 2
(a) 1 only (b) 2 only
(c) Both 1 and 2 (d) Neither 1 nor 2 Q.15 Which of the following is/are initiatives
taken by the government for increasing the
Q.11 Which of the following forms part of India’s flow of credit to agriculture sector in India?
external debt? 1. Kisan Credit Card
1. Commercial Borrowings 2. Farm Loan waiver
2. Trade Credit 3. Refinance Support
3. NRI Deposits 4. Interest subvention scheme
4. Rupee Debt Select the correct answer using the code given
below:
Select the correct answer using the code given (a) 1, 2 and 4 only (b) 1 and 4 only
below: (c) 2 and 4 only (d) 1, 2, 3 and 4
(a) 1 and 3 only (b) 1 and 2 only
Q.16 The Index of Industrial Production (IIP) is
(c) 3 and 4 only (d) 1, 2 3 and 4 one of the measure of industrial performance.
Q.12 With reference to share of agriculture in Gross Which of the following has highest weightage
Value Added (GVA) in Indian Economy, (use based classification) in the index
calculation?
consider the following statements:
(a) Primary Goods
1. There has been continuous growth in the (b) Consumer Durables
share of agriculture and allied sectors in (c) Capital Goods
GVA in last decade (d) Intermediate Goods
2. Falling share of agriculture and allied Q.17 With reference to Gross Capital formation
sectors in GVA is an expected outcome in (GCF) in Agriculture and allied sector,
a fast growing and structurally changing consider the following statements:
economy. 1. GCF in agriculture and allied sector has
consistently increased in last 5 years.
Which of the statements given above is/are 2. Laying of orchards and plantations is not
correct? included under GCF.
(a) 1 only (b) 2 only
Which of the statements given above is/are
(c) Both 1 and 2 (d) Neither 1 nor 2 correct?
Q.13 Which of the following can be the impact of (a) 1 only (b) 2 only
implementation of recommendations of the (c) Both 1 and 2 (d) Neither 1 nor 2
National Commission on Farmers (NCF) on Q.18 With reference to Pradhan Mantri Gramin
agriculture sector? Digital Saksharta Abhiyan, consider the
1. Higher growth in agricultural productivity following statements:
1. The Scheme is applicable for rural areas of
2. Prevention of farmer suicide
the country only.
3. Export of indigenous breeds and import of 2. Only such households where none of the
suitable breeds to increase productivity of family member is digitally literate will be
nondescript animals. eligible under the Scheme.
4. Inclusion of agriculutre in concurrent list. 3. Only one person per eligible household
would be considered for training
Select the correct answer using the code given
below: Which of the statements given above is/are
correct?
(a) 1 and 2 only (b) 1 and 4 only
(a) 1 only (b) 2 and 3 only
(c) 3 only (d) 1, 2, 3 and 4 (c) 1 and 3 only (d) 1, 2 and 3

© Copyright:   www.nextias.com

https://t.me/TheHindu_Zone_official
Join TelegramGroups
To Boost Your Preparation
PDF4Exams One stop solution for study
Click Here materials of all competitiveexams

The Hindu ZoneOfficial


Newspapers & study Click Here
materials

TestSeries4Exam
All paid test series
Click Here availabblewithoutanycost

Pdfbasket
All e-Magazines
in your hand Click Here
Hindi Books
All study materials
Click Here
in Hindi

eSandesh (An Indian App)

For More download eSandesh App from play store


https://t.me/IAS201819 https://t.me/PDF4Exams https://t.me/PDF4Exams

4 | CSE (Prelims) Test Series-2019


Q.19 Which of the following are interventions Q.23 Which of the following events coincided with
under Bharatmala Pariyojana- a new umbrella the period of the Great French Revolution?
program for the highways sector? 1. Introduction of Permanent Settlement
1. Development of Economic Corridors 2. Pitt’s India Act
2. Development of Border roads 3. First newspaper in India was started
3. Development of Coastal and Port 4. Sanskrit college at Benaras was established
connectivity roads by Jonathan Duncan
Select the correct answer using the code given
Select the correct answer using the code given
below:
below: (a) 2 and 3 only (b) 1 and 4 only
(a) 1 and 2 only (b) 1 and 3 only (c) 1, 2, 3 and 4 (d) 1, 3 and 4 only
(c) 2 and 3 only (d) 1, 2 and 3
Q.24 “He was a brilliant teacher who, in spite of
Q.20 With reference to Accredited Social Health his youth, attached to himself a host of bright
Activist (ASHA), consider the following and adoring students to think rationally
statements: and freely, to question all authority, to love
1. ASHA is trained to work as an interface liberty, equality and freedom, and to worship
between the community and the public truth. He was perhaps the first nationalist
health system poet of modern India.” The personality being
2. ASHA are selected for rural areas only. described here is:
3. ASHA is a depot holder for drug kits (a) Ishwar Chandra Vidyasagar
issued to Anganwadi worker. (b) Muhammad Iqbal
(c) Henry Vivian Derozio
Which of the statements given above is/are (d) David Hare
correct?
Q.25 With reference to Kingdom of Awadh in the
(a) 1 only (b) 3 only nineteenth century, consider the following
(c) 1, and 3 only (d) 1, 2 and 3 statements:
Q.21 Which of the following statements are 1. Nawabs of Awadh had been British foes
not correct with reference to Sikh reform since the Battle of Buxar.
movements in Punjab? 2. They were efficient rulers and taken a
1. Followers of Nirankari Movement wore number of measures for the welfare of its
people.
white clothes and gave up meat eating.
3. Annexation of Awadh was one of the
2. Namdhari movement emphasized on contributing factors for the Revolt of 1857.
worship of God as formless.
3. Singh Sabhas helped in setting up khalsa Which of the statements given above is/are
college at Amritsar in 1892. not correct?
(a) 1 and 2 only (b) 1, 2 and 3
Select the correct answer using the code given (c) 2 and 3 only (d) 3 only
below:
Q.26 The object of the Andrew Frazer Commission
(a) 1 only (b) 1 and 2 only
was
(c) 2 and 3 only (d) 3 only (a) To review the police administration
Q.22 With reference to administrative reforms (b) Indianization of British Army
introduced by Cornwallis, consider the (c) British expansion policies in North west
following statements: provinces.
1. For efficient administration he created (d) To prepare a code on Famines.
additional administrative posts. Q.27 With reference to the judicial system
2. The collectors were deprived of their established by British in India, consider the
judicial powers and their work were kept following statements:
limited to the collection of revenue. 1. British administration was largely carried
3. Disappointed with the corrupt practices in according to laws as interpreted by the
of the Company servants, he persuaded courts.
the Directors of the Company to penalize 2. Ilbert Bill succeeded in removing racial
them with huge cut in their salary. inequality between Indian and European
judges in courts.
Which of the statements given above is/are
Which of the statements given above is/are
correct? correct?
(a) 1 and 3 only (b) 2 only (a) 1 only (b) 2 only
(c) 1, 2 and 3 (d) 2 and 3 only (c) Both 1 and 2 (d) Neither 1 nor 2

© Copyright:   www.nextias.com

https://t.me/TheHindu_Zone_official
https://t.me/IAS201819 https://t.me/PDF4Exams https://t.me/PDF4Exams

Economy, Revision of Test-3 & Current Affairs_190904 | 5

Q.28 With reference to “Indian Parliamentary Q.32 With reference to the Anglo-French rivalry
Committee” established in Britain during the in trade and over the possessions in India,
initial phase of indian National Movement, consider the following statements:
consider the following statements: 1. The policy of securing monetary,
1. It was formed in the same year, the commercial or territorial favours from
‘Parliament of the World’s Religions’ took the victor by intervening in the internal
quarrels among local rulers was devised
place in Chicago.
by Robert Clive.
2. The purpose of the committee was to voice 2. The breach of “Treaty of Paris” led to the
India’s grievances in the Parliament of Battle of Wandiwash.
Britain.
Which of the statements given above is/are
Which of the statements given above is/are correct?
correct?: (a) 1 only (b) 2 only
(a) 1 only (b) 2 only (c) Both 1 and 2 (d) Neither 1 nor 2
(c) Both 1 and 2 (d) Neither 1 nor 2 Q.33 Arrange the places related to the following
Q.29 With reference to the Government of India establishments from north to south on India’s
Act ,1858, consider the following statements: map:
1. First Cotton textile mill of India
1. It created the post of Viceroy by abolishing
2. First factory opened by British East India
the post of Governor General of India. Company in India.
2. The Act was followed by the “Queen’s 3. Full fortified factory of French East India
Proclaimation of 1858’ through which Company.
Queen Victoria assumed a new title:
Select the correct answer using the code given
Qaisar-i Hind, the Empress of India.
below:
Which of the statements given above is/are (a) 2-1-3 (b) 2-3-1
correct? (c) 1-2-3 (d) 3-2-1
(a) 1 only (b) 2 only Q.34 With reference to the Charter Act of 1833,
(c) Both 1 and 2 (d) Neither 1 nor 2 consider the following statements:
1. For the first time British took the
Q.30 With reference to reorganisation of Army
responsibility for education of native
after the Revolt of 1857, consider the following
Indians and a sum of one lakh rupees was
statements: to be set aside for the same.
1. Discrimination on the basis of Caste and 2. The Act is popularly known as “Saint
religion was ended in the recruitment of Helena Act” provided for addition of a
army personnel. law member to the Governor General’s
2. Martial classes of army was to be consisted council.
of soldiers from Awadh, Bihar central 3. Slavery was abolished in India in 1833
India and South India. following the provisions made in this act.
3. The proportion of Europeans to Indians in Which of the statements given above is/are
the Bengal Army was fixed at two to one. correct?
(a) 1 and 3 only (b) 2 and 3 only
Which of the statements given above is/are
(c) 1 only (d) 2 only
not correct?:
(a) 1 and 2 only (b) 2 and 3 only Q.35 With reference to the Modern Indian History,
(c) 3 only (d) 1, 2 and 3 consider the following statements:
1. Wellesley adopted the “Policy of Ring
Q.31 With reference to treaties signed by British, fence” in wars against the Marathas and
Consider the following pairs: Mysore with an aim to create buffer zones
Treaty Participant to defend the Company’s frontier
1. Treaty of Yandabo Kingdom of Nepal 2. Hyderabad was the first state to accept the
2. Treaty of Gandmark Afghanistan subsidiary alliance with British.
3. Treaty of Salbai Maratha 3. Policy of annexation was abandoned by
the Queen’s proclamation of 1858.
Which of the pairs given above is/are
Which of the statements given above is/are
correctly matched?
correct?
(a) 1, 2 and 3 (b) 2 only
(a) 1 and 2 only (b) 2 and 3 only
(c) 2 and 3 only (d) 1 and 3 only (c) 1, 2 and 3 (d) 1 and 3 only

© Copyright:   www.nextias.com

https://t.me/TheHindu_Zone_official
https://t.me/IAS201819 https://t.me/PDF4Exams https://t.me/PDF4Exams

6 | CSE (Prelims) Test Series-2019


Q.36 With reference to the Indian Civil Services Q.40 The Lottery Committee’ of 1817 was associated
in British India, consider the following with the town planning of Kolkata.It was so
statements: called because
1. Fort William college for the training of (a) funds for city development were raised
new recruits by Cornwallis. through public lotteries.
2. Aitchison committee recommended direct (b) it was led by Lord Lottery
recruitment to ICS on basis of 50:50 pairity (c) its members were elected using lottery
between the Europeans and Indians. system
(d) its members represented in the British
3. Following the demand of Indian National
Parliament using lottery system
Conference, the House of Commons in
England in 1893, passed a resolution Q.41 With reference to the Constitution (123rd
supporting holdings of simultaneous Amendment) Bill, 2017, recently passed in
examination in India and England. Parliament, consider the following statements:
1. It seeks to grant the National Commission
Which of the statements given above is/are on Backward Classes (NCBC) constitutional
correct? status.
(a) 1 and 2only (b) 3 only 2. It seeks to remove the power of the National
(c) 2 and 3 only (d) None of these Commission for Scheduled Castes (NCSC)
to examine matters related to backward
Q.37 With respect to Arya Samaj which of the
classes and anglo-Indians.
following statement(s) is/are not correct? 3. National Commission on Backward Classes
1. The first formal unit of samaj was (NCBC) will comprise of five members
established in 1875 at Lahore. appointed by the President.
2. It followed the Vedic notion of Chaturvarna
system. Which of the statements given above is/are not
3. It strongly opposed the spread of Western correct?
(a) 2 only (b) 2 and 3
education and believed in the superiority
(c) 3 only (d) 1 and 2
of Vedas.
Q.42 With reference to Work Programme 2018-2020
Select the correct answer using the code given of Horizon 2020, recently in news, consider the
below: following statements:
(a) 1 and 3 only (b) 1, 2 and 3 1. Horizon 2020 is the largest multinational
(c) 2 only (d) 1 and 2 only programme dedicated to research and
Q.38 Which of the following statement(s) is/are innovation
correct? 2. It is open only to the developing nations of
the world
1. Guru Nanak rejected all orthodox religions
2. Guru Nanak compiled the compositions Which of the following statements given above
for Guru Granth Sahib in a new script is/are correct?
known as Gurmukhi (a) 1 only (b) 2 only
3. Some of the works of Kabirdas were (c) Both 1 and 2 (d) Neither 1 nor 2
collected and preserved in the Guru Q.43 The Government has been continuously
Granth Sahib working towards improving the ease of doing
Select the correct answer using the code given business across the country. Consequently, it
was announced that India will be developing
below:
a National Logistics Portal. Which of the
(a) 1 and 3 only (b) 1, 2 and 3
following statements is/are true regarding
(c) 2 only (d) 1 and 2 only National Logistics Portal?
Q.39 With reference to Young Bengal Movement, 1. The portal will link all the stakeholders of
consider the following statements : EXIM, domestic trade and movement and
1. It was led by Raja Rammohan Roy all trade activities on a single platform.
2. The inspiration drawn for the movement 2. It will ensure ease of trading in the domestic
was from the idea of “natural rights” of markets only.
3. It will be developed by the Ministry of
American Revolution.
Finance
Which of the statements given above is/are
Select the correct answer using the code given
correct? below:
(a) 1 only (b) 2 only (a) 1 only (b) 2 and 3 only
(c) Both 1 and 2 (d) Neither 1 nor 2 (c) 2 only (d) 1, 2 and 3

© Copyright:   www.nextias.com

https://t.me/TheHindu_Zone_official
https://t.me/IAS201819 https://t.me/PDF4Exams https://t.me/PDF4Exams

Economy, Revision of Test-3 & Current Affairs_190904 | 7

Q.44 Which of the following products in India has 2. Minimizing nitrogen’s negative effects on
got Geographical Indication Tag? both, human health and the environment
1. Alphonso Mango of Maharashtra
2. Shahi litchi of Bihar Select the correct answer using the code given
3. Basmati rice of Madhya Pradesh below
4. Kadaknath Chicken of Madhya Pradesh (a) 1 only (b) 2 only
(c) Both 1 and 2 (d) Neither 1 nor 2
Select the correct answer using the code given
below: Q.49 Khangchendzonga has been in news recently.
(a) 2, 3 and 4 only (b) 1, 3 and 4 only Which of the following is/are correct
(c) 1, 2 and 4 only (d) 1, 2 and 3 only regarding Khangchendzonga?
Q.45 With reference to Ballistic Missile Defence 1. Khangchendzonga National Park (KNP)
System of India (BMD system), consider the has an unsurpassed range of sub-tropical
following statements: to alpine ecosystems.
1. It is developed by Indian Space Research 2. Khangchendzonga National Park (KNP),
Organization (ISRO)
has been inscribed as India’s first “Mixed
2. For low altitude interception, the BMD
system uses Prithvi Air Defence (PAD) World Heritage Site” on UNESCO World
missile Heritage List.
3. For high altitude interception, the BMD 3. Khangchendzonga Biosphere Reserve was
system uses Advanced Air Defence (AAD) the first Biosphere Reserve from India
Missile to be Included in the World Network of
Which of the statements given above is/are not Biosphere Reserves
correct? Select the correct answer using the code given
(a) 1 only (b) 1 and 2 only below
(c) 3 only (d) 1, 2 and 3
(a) 1 and 2 only (b) 2 only
Q.46 Recently, India dedicated the laboratory for the (c) 1 and 3 only (d) 1, 2 and 3
conservation of endangered species (LaCONES)
in Hyderabad. With reference to LaCONES, Q.50 With reference to the Asia Pacific Institute of
consider the following statements: Broadcasting Development (AIBD), consider
1. LaCONES is the only laboratory in India the following statements
that has developed methods for collection 1. It is a regional inter-governmental
and cryopreservation of semen, eggs and organisation servicing countries of the
embryos of endangered species United Nations Economic and Social
2. It would facilitate exchange of genetic Commission for Asia and the Pacific (UN-
material between the Indian zoos for
ESCAP) in the field of electronic media
maintaining genetic diversity.
development.
Which of the statements given above is/are not 2. India has been elected as the President
correct? of Asia-Pacific Institute for Broadcasting
(a) 1 only (b) 2 only
Development (AIBD) for the first time
(c) Both 1 and 2 (d) Neither 1 nor 2
Q.47 ”3D printing” has applications in which of the Which of the following statements given
following? above is/are correct?
1. Coral Reefs (a) 1 only (b) 2 only
2. Lithium-ion batteries (c) Both 1 and 2 (d) Neither 1 nor 2
3. Reconstructive surgeries
Q.51 Which of the following advantages were
Select the correct answer using the code given provided by the ‘Treaty of Sagauli of 1816’ to
below: the British?
(a) 2 only (b) 3 only 1. It facilitated trade with Central Asia.
(c) 1 and 2 only (d) 1, 2 and 3
2. It helped in enhancing British influence in
Q.48 Recently, India’s N Raghuram has been elected Garhwal and Kumaon region.
Chair of the International Nitrogen Initiative
(INI), a global policy making initiative. Which Select the correct answer using the code given
of the following is/are the objectives of INI? below
1. Optimizing nitrogen’s beneficial role in (a) 1 only (b) 2 only
sustainable food production (c) Both 1 and 2 (d) Neither 1 nor 2

© Copyright:   www.nextias.com

https://t.me/TheHindu_Zone_official
https://t.me/IAS201819 https://t.me/PDF4Exams https://t.me/PDF4Exams

8 | CSE (Prelims) Test Series-2019


Q.52 Who among of the following signed the Select the correct answer using the code given
‘Treaty of Lahore of 1838’? below
1. Maharaja Ranjit Singh (a) 1 and 3 (b) 1, 3 and 4
2. Lord Lytton (c) 2, 3 and 4 (d) 1, 2 and 3
3. Dost Mohammed Q.56 Consider the following statements
Select the correct answer using the code given 1. By the Charter Act, 1813 debts of the
below Company were taken over by the
(a) 1 only (b) 1 and 2 Government of India which was also to
(c) 2 and 3 (d) 1 and 3 pay its shareholders a dividend on their
capital
Q.53 Over the course of time which of the following 2. Charter Act of 1833, allowed Christian
steps had been adopted by the British to missionaries to enter India without
prevent Afghanistan’s inclination towards restrictions.
Russia?
Which of the statements given above is/are
1. Policy of Non-interference
correct?
2. Occasional Help
(a) 1 only (b) 2 only
3. Armed Invasion (c) Both 1 and 2 (d) Neither 1 nor 2
Select the correct answer using the code given Q.57 The Indian Currency Committee appointed
below by the British Indian Government to examine
(a) 1 and 2 (b) 2 and 3 the currency situation in India is popularly
(c) 1 and 3 (d) 1, 2 and 3 known as
Q.54 Consider the following statements: (a) Fowler Committee
1. Lord Lytton was the Viceroy when Indian (b) Hunter Committee
National Congress was formed in 1885 at (c) Shore Committee
Bombay. (d) Raleigh Commission
2. George Yule became the first British Q.58 With reference to the Anglo-Burmese
President of INC and presided over the relations, consider the following:
Allahabad session in 1888. 1. Treaty of Yandaboo was signed between
3. Maulana Abul Kalam Azad, the Bharat the British and Burma during the second
Ratna, was the Congress President from Anglo-Burmese war.
1940-46. 2. The First Burmese War was in part the
result of border clashes while the Second
Which of the statements given above is/are Burmese War was almost wholly the result
correct? of British commercial greed.
(a) 1 and 3 (b) 2 and 3 3. In 1935, the British step of separating
(c) 3 only (d) 1 only Burma from India in the hope of weakening
Q.55 ‘The reactionary administration of Lord the Burmese struggle for freedom was
Lytton had aroused the public from its attitude opposed by the Burmese nationalists.
of indifference and had given a stimulus Which of the statements given above is/are
to public life. In the evolution of political correct?
progress, bad rulers are often a blessing in (a) 1 and 2 (b) 2 only
disguise. They help to stir a community into (c) 2 and 3 (d) 1 and 3
life, a result that years of agitation would
Q.59 Consider the following statements:
perhaps have foiled to achieve.’
1. Peel Commission recommended
Which of the following actions is talked about the native army to be composed of
in the passage? different nationalities and castes, mixed
1. Removal of import duty on British textile promiscuously through each regiment.
imports 2. The Indian States Committee formed in
2. The Indian Official Secrets Act was passed 1928 under Sir Harcourt Butler, reaffirmed
restricting the freedom of the press. the supremacy of paramountcy but only
3. Holding the imperial Durbar at Delhi with limited power.
when country was suffering from famine Which of the statements given above is/are
4. A new regulations reducing the maximum correct?
age limit for sitting in the Indian Civil (a) 1 only (b) 2 only
Service Examination from 21 years to 19. (c) Both 1 and 2 (d) Neither 1 nor 2

© Copyright:   www.nextias.com

https://t.me/TheHindu_Zone_official
https://t.me/IAS201819 https://t.me/PDF4Exams https://t.me/PDF4Exams

Economy, Revision of Test-3 & Current Affairs_190904 | 9

Q.60 With reference to the ‘Treaty of Gandamak’, Select the correct answer using the code given
consider the following statements below:
1. It was signed after the first Anglo-Afghan (a) 1, 2 and 3 (b) 2 and 3
war during the reign of Lord Auckland. (c) 1 and 2 only (d) 1 and 3
2. British gained control over Afghan’s foreign Q.65 Which of the following statements about State
policy Disaster Relief Fund(SDRF) is/are true?
Which of the statements given above is/are 1. SDRF is located in the ‘Consolidated Fund
correct? of State’ under ‘Reserve Fund’
(a) 1 only (b) 2 only 2. Government of India (GoI) contributes
(c) Both 1 and 2 (d) Neither 1 nor 2 75% of the SDRF of the general category
states and 90% of the special category
Q.61 With reference to FDI in India in the last five states.
years, consider the following statements: 3. SDRF shall be used only for meeting the
1. Singapore was the top source of FDI in India expenditure for providing immediate
in 2017-18. relief to the victims
2. India is among top 5 economies in terms of
Select the correct answer using the code given
FDI recieved annually. below:
Which of the statements given above is/are (a) 1 only (b) 1 and 2 only
correct? (c) 3 only (d) 2 and 3 only
(a) 1 only (b) 2 only Q.66 With reference to Agmark, a certification
(c) Both 1 and 2 (d) Neither 1 nor 2 mark employed on agricultural products in
Q.62 Which of the following schemes have been India, consider the following statements:
subsumed under National Health Protection 1. It conform to a set of standards approved
Scheme? by the Food Safety and Standards
1. Rashtriya Swasthya Bima Yojana (RSBY) Authority of India
2. Senior Citizen Health Insurance Scheme 2. It helps to cover the quality guidelines in
(SCHIS). agricultural products excluding pulses
and fresh fruits.
3. Pradhan Mantri Jeevan Jyoti Bima Yojana
Which of the statements given above is/are
Select the correct answer using the code given
correct?
below
(a) 1 only (b) 2 only
(a) 1 only (b) 1 and 2 only
(c) Both 1 and 2 (d) Neither 1 nor 2
(c) 1 and 3 only (d) 1, 2 and 3
Q.67 With reference to recently test fired ASTRA,
Q.63 The Indian Ocean Rim Association (IORA) is an consider the following questions:
international organisation consisting of coastal 1. It is a Short range air-to-air missile
states bordering the Indian Ocean. Which of the 2. It is developed in collaboration with Israel
following countries are not members of IORA? 3. It has an official range of 75 kilometers
1. Pakistan
2. Myanmar Which of the statements given above is/are
3. Maldives correct?
4. Bangladesh (a) 2 only (b) 1 and 3 only
(c) 2 and 3 only (d) 1, 2 and 3
Select the correct answer using the code given
Q.68 With reference to World Wildlife Foundation’s
below
(WWF) ‘Tx2’ programme, consider the
(a) 4 only (b) 1, 2 and 3 only
following statements:
(c) 2 and 4 only (d) 1, 2, 3 and 4
1. A global governmental goal by all Tiger
Q.64 Which of the following statements about Range Countries to double the number of
Financial Inclusion Index is/are true? wild tigers by 2022
1. It will be launched by Ministry of Commerce 2. According to a recent report of WWF,
and Industry. Bangladesh is set to become the first
2. It includes savings, remittances, credit, country to achieve the target of doubling
insurance and pension products of tiger population.
3. The index inter-alia will have two Which of the statements given above is/are
measurement dimensions- Access to correct?
financial services and Usage of financial (a) 1 only (b) 2 only
services (c) Both 1 and 2 (d) Neither 1 nor 2

© Copyright:   www.nextias.com

https://t.me/TheHindu_Zone_official
https://t.me/IAS201819 https://t.me/PDF4Exams https://t.me/PDF4Exams

10 | CSE (Prelims) Test Series-2019


Q.69 Which of the following statements about Q.73 Which among the following statements are
None Of The Above (NOTA) are true in true about International Criminal Court(ICC)?
context of elections in India? 1. It is governed by an international treaty
1. It was first introduced in elections in 2013. called Rome Statute.
2. It is same as right to reject. 2. India is founding member of International
3. NOTA is avaialble as an option in all
Criminal Court (ICC)
elections,including elections to Rajya Sabha.
3. It does not have the authority to investigate
Select the correct answer using the code given war crimes.
below:
(a) 1 only (b) 1 and 2 only Select the correct answer using the code given
(c) 1 and 3 only (d) 1, 2 and 3 below
(a) 1 only (b) 1 and 3 only
Q.70 Which of the following are the components of
(c) 2 and 3 only (d) 1, 2 and 3
recently launched “Pradhan Mantri Annadata
Aay SanraksHan Abhiyan” (PM-AASHA)? Q.74 With reference to Central Pollution Control
1. Price Support Scheme (PSS) Board, consider the following statements:
2. Accelerated Irrigation Benefit Program 1. It is a statutory organisation constituted
3. Price Deficiency Payment Scheme (PDPS) in 1974 under the Air (Prevention and
4. Private Procurement and Stockist Scheme
Control of Pollution) Act, 1974.
(PPPS).
2. One of the mandates of CPCB is to collect,
5. National Agriculture Insurance Scheme
collate and disseminate technical and
Select the correct answer using the code given statistical data relating to water pollution.
below:
(a) 1, 4 and 5 (b) 1, 2 and 3 Which of the statements given above is/are
(c) 2, 3 and 4 (d) 1, 3 and 4 correct?
(a) 1 only (b) 2 only
Q.71 With reference to National Nutrition Mission
(c) Both 1 and 2 (d) Neither 1 nor 2
(POSHAN Abhiyaan), consider the following
statements: Q.75 With reference to Akash missiles, consider
1. It is a flagship scheme of Ministry of Health the following statements:
and Family Welfare. 1. It is short range surface-to-air missile
2. The objective of POSHAN Abhiyaan is to 2. The missile is developed under the
reduce stunting in all Districts of India
integrated guided-missile development
3. It aims at improving utilization of key
Anganwadi Services and improving the programme (IGMDP).
quality of Anganwadi Services delivery. 3. It can engage multiple targets at a time in
all-weather conditions
Which of the statements given above is/are
correct? Which of the statements given above is/are
(a) 2 and 3 only (b) 1 only correct?
(c) 1, 2 and 3 (d) 3 only (a) 2 only (b) 2 and 3 only
Q.72 With reference to recently signed security (c) 1 and 3 only (d) 1, 2 and 3
agreements between US and India, consider the Q.76 Which of the following provisions relate to
following statements: the recent Supreme Court judgement about
1. Logistics Exchange Memorandum of Aadhar?
Agreement (LEMOA) was signed at recent
1. Statutory bodies like CBSE and UGC can
2+2 dialogues
ask students to produce their Aadhaar
2. India has signed three out of four
foundational security agreements with US cards for examinations like NEET and JEE.
3. Communications Compatibility and 2. Children, once they attain the age of
Security Agreement (COMCASA) is to maturity, could opt out of Aadhaar.
facilitate access to advanced defence systems 3. Authentication records should not be
and enable India to optimally utilise its retained for more than six months.
existing U.S.-origin platforms.
Select the correct answer using the code given
Which of the statements given above is/are below:
correct? (a) 1, 2 and 3 (b) 1 only
(a) 1, 2 and 3 (b) 1 and 2 (c) 1 and 2 only (d) 2 and 3 only
(c) 2 and 3 only (d) 2 only

© Copyright:   www.nextias.com

https://t.me/TheHindu_Zone_official
https://t.me/IAS201819 https://t.me/PDF4Exams https://t.me/PDF4Exams

Economy, Revision of Test-3 & Current Affairs_190904 | 11

Q.77 Which of the following steps taken by Q.81 In Union Budget 2017-18, government
government would help in stabilising rupee? has announced to award “affordable
1. Efforts to reduce non-essential imports. housing” infrastructure status. Which of the
2. Attracting more Foreign Portfolio following body releases Harmonized List of
Investors (FPI) into the corporate debt Infrastructure Sub-sectors?
(a) Department of Economic Affairs
market.
(b) Department of Commerce
3. Decreasing exports. (c) Department of Financial Service
4. Issuing more masala bonds (d) Department of Investment and Public
Select the correct answer using the code given Asset Management
below: Q.82 The term “Baltic Dry Index” relates to:
(a) 1, 2 and 3 only (b) 1 and 4 only (a) Crude Oil Price
(c) 1, 2 and 4 only (d) 1, 2, 3 and 4 (b) Shipping and Trade
(c) Income inequality
Q.78 With reference to recently launched India (d) Price of Cereals.
Post Payment Bank, consider the following
statements Q.83 With reference to real estates in India,
1. It will reduce the opportunity cost of consider the following statements:
1. As per Economic Survey 2017-18, real
unbanked population.
estate and construction together, is the
2. It will provide person to person second largest employment provider in
remittance services including cross border the country.
remittances. 2. RESIDEX, India’s first official housing
3. It is exempted from meeting Statutory price index is released by Bombay Stock
Liquidity Ratio requirement of Reserve Exchange.
Bank of India.
Which of the statements given above is/are
Which of the statements given above is/are correct?
correct? (a) 1 only (b) 2 only
(a) 1 only (b) 1 and 2 only (c) Both 1 and 2 (d) Neither 1 nor 2
(c) 2 and 3 only (d) 1, 2 and 3 Q.84 With reference to Pradhan Mantri Jan
Arogya Yojana (PM-JAY), which of following
Q.79 Consider the following statements:
statement is not correct?
1. Gross Enrolment Ratio is the ratio of (a) It seeks to accelerate India’s progress
children of the official primary school age towards achievement of Universal Health
who are enrolled in primary school to the Coverage (UHC).
total population of the official primary (b) It will cover medical and hospitalization
school age. expenses for primary, secondary and
2. Net Enrolment Ratio is the student tertiary care procedures.
enrolment as a proportion of the (c) There will be no cap on family size and age
corresponding eligible age group in a in the Mission
given year. (d) The beneficiaries identified under SECC,
2011 will not be required to pay any
Which of the statements given above is/are charges for the hospitalization expenses
correct?
Q.85 With reference to National Free Drug
(a) 1 only (b) 2 only Initiative, consider the following statements:
(c) Both 1 and 2 (d) Neither 1 nor 2 1. It is an initiative under National Health
Q.80 With reference to Global Counterterrorism Mission.
Forum(GCTF), consider the following 2. It aims to expand the availability of
statements: free drug provision in all public health
facilities.
1. GCTF is an informal multilateral
3. Union Government will be solely
counterterrorism platform launched by responsible for purchase and distribution
United Nation Resolution in 2006 of drugs.
2. Both India and Pakistan are members of 4. It will help reducing the high out of pocket
GCTF expenditure.
Which of the statements given above is/are Which of the statements given above is/are
correct? correct?
(a) 1 only (b) 2 only (a) 1 and 3 only (b) 2 and 4 only
(c) Both 1 and 2 (d) Neither 1 nor 2 (c) 1, 2 and 4 only (d) 1,2 3 and 4

© Copyright:   www.nextias.com

https://t.me/TheHindu_Zone_official
https://t.me/IAS201819 https://t.me/PDF4Exams https://t.me/PDF4Exams

12 | CSE (Prelims) Test Series-2019


Q.86 With reference to expenditure of general 3. In a depressed industry, structural
government (Centre and States), consider the unemployment takes place on account of
following statements: change in the supply pattern
1. Annual Expenditure of general Which of the statements given above is/are
government in health has been more than correct?
education. (a) 1 and 2 only (b) 1 and 3 only
2. Total Expenditure of general government (c) 2 and 3 only (d) 1, 2 and 3 only
in social services has successively
increased since 2012-13. Q.91 With reference to Mission on Integrated
Development of Horticulture (MIDH),
Which of the statements given above is/are consider the following statements:
correct? 1. It aims for holistic development of
(a) 1 only (b) 2 only horticulture sector in India including
(c) Both 1 and 2 (d) Neither 1 nor 2 bamboo.
Q.87 With reference to UNDP-Human 2. National Horticulture Mission is a sub-
scheme of MIDH.
Development Index, consider the following
3. It will promote development of Micro-
statements:
Irrigation for all horticulture crops.
1. Among BRICS nations, India ranks last in
HDI ranking in the latest report. Which of the statements given above is/are
2. Human Development Index value of India correct?
has consistently improved. (a) 1 and 2 only (b) 2 and 3 only
(c) 1 and 3 only (d) 1, 2 and 3
Which of the statements given above is/are
correct? Q.92 Which of the following is/are non-tariff trade
(a) 1 only (b) 2 only barriers?
(c) Both 1 and 2 (d) Neither 1 nor 2 1. Subsidies for domestic goods
2. Foreign exchange restrictions
Q.88 With reference to subsidy given by Union 3. Domestic content requirements
government in India, consider the following 4. Industry bailouts
statements:
1. Food Subsidy forms the largest part of Select the correct answer using the code given
subsidy given by government. below:
(a) 2 and 3 only (b) 1, 2 and 3 only
2. Subsidy on petroleum is more than
(c) 1 and 4 only (d) 1, 2, 3 and 4
fertiliser.
Q.93 Consider the following statements:
Which of the statements given above is/are 1. India was the highest producer of rice in
correct? world, followed by china, in 2017.
(a) 1 only (b) 2 only 2. India is the second highest producer of
(c) Both 1 and 2 (d) Neither 1 nor 2 sugar, after Brazil, in 2017.
Q.89 Consider the following pairs: Which of the statements given above is/are
Index Released by correct?
1. Wholesale : DIPP (a) 1 only (b) 2 only
Price Index (c) Both 1 and 2 (d) Neither 1 nor 2
2. Consumer Price : CSO
Index (Industrial Worker) Q.94 With reference to Development Box under
3. Purchasing Manager : CSO World Trade Organization(WTO), consider
Index (PMI) the following statements:
4. Index of Industrial : Labour 1. It is ‘amber box with conditions’ designed
to reduce trade distortion
Production Bureau
2. Only the direct measures of assistance that
Which of the pairs given above is/are are an integral part of the development
correctly matched? programmes of developing countries falls
(a) 1 only (b) 1 and 3 only under this category
(c) 1 and 2 only (d) 1, 2, 3 and 4 3. Agricultural input subsidies generally
available to low-income developing
Q.90 With reference to Structural unemployment, countries are included in this category.
consider the following statements:
1. It is caused by a decline in demand for Which of the statements given above is/are
production in a particular industry correct?
2. It is a natural concomitant of economic (a) 1 and 2 only (b) 3 only
progress and innovation (c) 1 and 3 only (d) 1, 2 and 3 only

© Copyright:   www.nextias.com

https://t.me/TheHindu_Zone_official
https://t.me/IAS201819 https://t.me/PDF4Exams https://t.me/PDF4Exams

Economy, Revision of Test-3 & Current Affairs_190904 | 13

Q.95 With reference to World Bank, consider the Q.98 Prime Minister Krishi Sinchayee
following statements: Yojana(PMKSY) has been formulated
1. World Bank give loans for both policy amalgamating various ongoing schemes.
reforms and development projects Which of the following schemes are not under
2. The term “World Bank” refers only to PMKSY?
1. Accelerated Irrigation Benefit Programme
International Bank for Reconstruction and
(AIBP)
Development (IBRD) and International
2. Command Area Development Programme
Development Association (IDA) 3. Drought Prone Areas Programme
Which of the statements given above is/are Select the correct answer using the code given
correct? below
(a) 1 only (b) 2 only (a) 3 only (b) 1 and 2 only
(c) Both 1 and 2 (d) Neither 1 nor 2 (c) 2 and 3 only (d) 1, 2 and 3
Q.96 New Urea Policy (NUP) 2015 seeks to: Q.99 With reference to Gender Development Index
1. increase indigenous urea production (GDI), consider the following statements:
2. promote energy efficiency in urea 1. It uses the same component indicators as
production used in the HDI
3. reduce subsidy burden on the central 2. It is released by UN Women, United
government. Nations entity dedicated to gender
equality and the empowerment of women
Which of the statements given above is/are
Which of the statements given above is/are
correct?
correct?
(a) 1 and 2 only (b) 3 only (a) 1 only (b) 2 only
(c) 1 and 3 only (d) 1, 2 and 3 only (c) Both 1 and 2 (d) Neither 1 nor 2
Q.97 Which of the following is/are the Sustainable Q.100 With reference to the World Trade
Development Goals framed by United Organization (WTO), consider the following
Nations Development Programme Network. statements:
1. Partnerships for the goals 1. It is the only global international
2. Industry, innovation and infrastructure organization dealing with the rules of
3. Life Below Water trade between nations.
4. Life on land 2. Decisions in the WTO are generally taken
5. Responsible consumption and production by consensus of the entire membership
3. ‘Most Favoured Nation’ (MFN) status
Select the correct answer using the code given under WTO denotes the equal treatment
below. of all countries.
(a) 2, 3, 4 and 5 only Which of the statements given above is/are
(b) 1, 2 and 3 only correct?
(c) 2 and 5 only (a) 2 and 3 only (b) 1 and 2 only
(d) 1, 2, 3, 4 and 5 (c) 3 only (d) 1, 2 and 3 only
nnnn

© Copyright:   www.nextias.com

https://t.me/TheHindu_Zone_official
https://t.me/IAS201819 https://t.me/PDF4Exams https://t.me/PDF4Exams

14 | CSE (Prelims) Test Series-2019

SPACE FOR ROUGH WORK

© Copyright:   www.nextias.com

https://t.me/TheHindu_Zone_official
https://t.me/IAS201819 https://t.me/PDF4Exams https://t.me/PDF4Exams

Economy, Revision of Test-3 & Current Affairs_190904 | 15

SPACE FOR ROUGH WORK

© Copyright:   www.nextias.com

https://t.me/TheHindu_Zone_official
https://t.me/IAS201819 https://t.me/PDF4Exams https://t.me/PDF4Exams

16 | CSE (Prelims) Test Series-2019

BIG LEARNINGS MADE E ASY

An initiative of Group

17
Books

Modern Indian History


History of Medieval India

History of Ancient India

History of the World

Indian Economy
Internal Security
Indian Society
Post-Independence History
International Relations

Science & Technology


Physical & World Geography
& Disaster Management

Ecology and Environment


Indian Geography

Life Sciences

Indian Polity

Ethics

Corporate Office: 44A/1, Kalu Sarai, New Delhi-110016 | 011-45124660, 08860378007


© MADE
Copyright: at leading book stores across India and on major e-comerce sites like:
EASY books are available   www.nextias.com

https://t.me/TheHindu_Zone_official
https://t.me/IAS201819 https://t.me/PDF4Exams https://t.me/PDF4Exams

Economy, Revision of Test-3 & Current Affairs_190904 | 1

CSE (Prelims) Test Series-2019 | Test-4


Detailed Explanations

1. (c) 5. (a)
Statement 1 is not correct: MSP is not statutorily World Bank publishes the “Report on migration
enforceable. and Remittances”. The report highlights the
Statement 2 is not correct: The minimum support emerging trends or topical issues, and are issued
prices are announced by the Government of every second month (or more frequently if the
occasion demands)
India at the beginning of the sowing season for
certain crops 6. (d)
Statement 3 is correct: The FRP set by the Centre Statement 1 is not correct: India exports more
is the minimum price that sugarcane farmers are engineering goods than electronics
legally guaranteed. Statement 2 is not correct: India’s imports consist
of more petroleum producets followed by gems
2. (d)
and jewellery
Statement 1 is not correct: The labour force
participation rate is calculated by expressing Export Share (per cent)
the number of persons in the labour force as Rank Items 2015-16 2016-17
a percentage of the working-age population. Engineering
The labour force is the sum of the number of 1 23.1 24.4
goods
persons employed and the number of persons
Gems and
unemployed 2 15.0 15.7
Jewellery
Statement 2 is not correct: The rural FLFPR is
Petroleum
significantly higher than urban FLFPR. 3 crude & 11.7 11.4
3. (b) products
The World Population Prospects is the official Electronic
4 2.2 2.1
United Nations population estimates and goods
projections that have been prepared by the 7. (c)
Population Division of the Department of
WTO’s Agreement on Agriculture (AoA)
Economic and Social Affairs of the United
classifies domestic support or subsidies given
Nations Secretariat
by the government to farmers into different
4. (c) categories. An important type of subsidies or
supports is Aggregate Measurement of Support
Statement 1 is correct: The current account
(AMS). The AMS represents trade distorting
deficit is a measurement of a country’s trade
domestic support and is referred as the “amb
where the value of the goods and services it
imports exceeds the value of the goods and 8. (d)
services it exports. A nation has a trade deficit if Statement 1 is not correct: The Logistics
the total value of goods and services it imports Performance Index is reported by the World
is greater than the total value of those it exports. Bank in every two years. Hence, Statement 1 is
Most nations borrow from foreign states to pay not correct.
for the imports. Therefore, a country with a trade Statement 2 is not correct: India’s ranking 2014:
deficit will also have a current account deficit. 54 rank, 2016:35 rank and in 2018 India slipped
Statement 2 is correct: The balance of trade to 44 rank. Thus India’s ranking has not steadily
influences currency exchange rates through its improved
effect on the supply and demand for foreign
9. (d)
exchange. When a country’s trade account does
Statement 1 is correct: PMGSY is a nationwide
not net to zero – that is, when exports are not
plan in India to provide good all-weather road
equal to imports – there is relatively more supply
connectivity to unconnected villages. The
or demand for a country’s currency, which
PMGSY is under the authority of the Ministry
influences the price of that currency on the world
of Rural Development and was begun on 25
market. December 2000.

© Copyright:   www.nextias.com

https://t.me/TheHindu_Zone_official
https://t.me/IAS201819 https://t.me/PDF4Exams https://t.me/PDF4Exams

2 | CSE (Prelims) Test Series-2019


Statement 2 is correct: It is fully funded by subsidy required for this would be one percent
the central government Road safety aspects of the Gross Domestic Product. Arrangements
have to be incorporated as part of the design for MSP need to be put in place for crops other
criteria during the development of Detailed than paddy and wheat. Also, millets and other
Project Report (DPR) for the Rural Road Project. nutritious cereals should be permanently
Application of the PMGSY standard in rural included in the PDS.
roads development should be accompanied with
sufficient knowledge of road safety requirements. 14. (a)
Statement 3 is correct: Geospatial rural road Statement 1 is correct: MPI assesses poverty at the
information system(GRRIS) is a geographical individual level.
presentation of PMGSY. It provides locational Statement 2 is not correct: The global MPI was
information about habitations, markets and developed by Oxford Poverty and Human
administrative headquarters. Development Initiative (OPHI) with the UN
Development Programme (UNDP) for inclusion in
10. (d)
UNDP’s flagship Human Development Report in
Statement 1 is not correct: The centrally 2010. It has been published in the HDR ever since
sponsored scheme aims at providing strategic
funding to higher educational institutions 15. (d)
throughout the country. Funding is provided All the given arrangements will help in increasing
by the central ministry through the state the flow of credit to the agriculture sector.
governments and union territories (UT). Arrangement 1 is correct: The Kisan Credit Card
Statement 2 is not correct: The Ministry of Scheme was first introduced in 1998 in order to
Human Resource Development (MHRD) has provide easy flow of credit and enables farmers to
constituted a National Mission Authority with purchase agricultural inputs consumption needs.
chairman -Union Human Resource Development Arrangement 2 is correct: Farm Loan waiver is the
Minister. practice whereby the Centre or States take over the
11. (d) liability of farmers and repay the banks.
External debt comprises of multilateral, bilateral Arrangement 3 is correct: Refinance Support is
trade credit, IMF, commercial borrowings ,NRI kind of mechanism to support institutional lender
deposits, rupee debt and short term debt. Hence, such as banks, NBFCs etc undertaken by special
all the given statements are correct. agency such as MUDRA.
Arrangement 4 is correct: Interest subvention
12. (b)
scheme offer Loan facility at a subsidized interest
Statement 1 is not correct: The growth in GVA rate, where the difference in actual interest rate and
isn’t continuously increasing For example it was subsidized interest rate is bear by the government.
5.2% in 2013-14 and 0.2% in 2014-15.
Statement 2 is correct: Falling share of agriculture 16. (a)
and allied sectors in GVA is an expected outcome According to new series of IIP, use based
in a fast growing and structurally changing classification has following wieghtage.
economy. Primary Goods has highest weightage of 34.05 %.
13. (d) Consumer Durables has weightage of 12.84%
The National Commission on Farmers (NCF) Capital Goods has weightage of 8.22%
was constituted on November 18, 2004 under the Intermediate Goods has weightage of 17.22%
chairmanship of Professor M.S. Swaminathan.
17. (d)
Some of the main recommendations include:
Distribute ceiling-surplus and waste lands; Statement 1 is not correct:
Prevent diversion of prime agricultural land and Share of Agriculture & Allied Sectors GCF in total
forest to corporate sector for non-agricultural Gross Capital Formation at current prices
purposes. Establish a National Land Use
2012-13 2013-14 2014-15 2015-16
Advisory Service set up a mechanism to regulate
the sale of agricultural land, based on quantum 7.7 9.0 8.3 7.8
of land, nature of proposed use and category of Statement 2 is not correct: orchards and plantations
buyer. Implement a universal public distribution form an essential component of the agriculture and
system. The NCF pointed out that the total allied sector.

© Copyright:   www.nextias.com

https://t.me/TheHindu_Zone_official
https://t.me/IAS201819 https://t.me/PDF4Exams https://t.me/PDF4Exams

Economy, Revision of Test-3 & Current Affairs_190904 | 3

18. (d) Statement 3 is not correct: Anganwadi Worker


Statement 1 is correct: Only rural areas are covered is a depot depot holder for drug kits and will be
under the Pradhan Mantri Gramin Digital Saksharta issuing it to ASHA.
Abhiyan to have a focused targeted approach. 21. (B)
Statement 2 and 3 are correct: Statement 1 is not correct: Nirankari Movement
Eligibility Creteria for PMDIGSHA: was founded by Baba Dayal Das and was
1. All such households where none of the family considered to be a movement of purification and
member is digitally literate will be considered return. The main focus behind the movement
eligible household under this scheme. was on deficiencies in religious practice.
According to them the appropriate path to reach
2. The benificiary should be digitally illiterate and God was through worship based on meditation
only one person per eligible household would rather than complex rituals and rites. Baba
be considered for training Dayal insisted the worship of God as nirankar
Additional Information: (formless).
Pradhan Mantri Gramin Digital Saksharta Statement 2 is not correct: The Namdhari
Abhiyan’ (PMGDISHA) envisaged to make 6 crore Movement was founded by Baba Ram Singh.
rural households digitally literate by March,.2019. His followers wore white clothes and gave up
This scheme will be implemented by Ministry of meat eating.The Namdharis were also known
Electronics and Information and Technology and is as “Kukas” because of their trademark style of
applicable for rural areas of the country. reciting the “Gurbani” (Sayings/Teachings of the
Guru). The Kukas played significant role in Non-
Priority would be given to non smartphone users, Cooperation and Civil Disobedience movement.
antyodaya household, and college dropouts
Statement 3 is correct: Singh Sabhas started in
Preference would be given to SC, ST, Women, BPL Lahore and Amritsar in 1870s were aimed at
and Minorities. reforming the Sikh society. They helped to set up
19. (d) the Khalsa College at Amritsar in 1892.They also
encouraged Gurmukhi and Punjabi literature.
Intervention 1 is correct: Bharatmala programme
has identified around 26,200 km of Economic 22. (B)
Corridors or routes that have heavy freight traffic Statement 1 is not correct: To cut down on
Intervention 2 is correct: Bharatmala also envisages extravagances, he abolished a number of surplus
building 3300 kms of Border Roads of strategic posts.
importance along international boundaries and Statement 2 is correct: He introduced the
2000 km of International Connectivity roads to separation of the three branches of service,
promote trade with Nepal, Bhutan, Bangladesh namely commercial, judicial and revenue. The
and Myanmar collectors, the king-pins of the administrative
system were deprived of their judicial powers
Intervention 3 is correct: In addition to the above,
and their work became merely the collection of
around 2,100 km of coastal roads and 2000 km of
revenue.
port connectivity roads have been identified under
Bharatmala. Statement 3 is not correct: Cornwallis was
determined to purify the administration, but he
Additional Information:
realised that the Company’s servants would not
Apart from the above corridors Bharatmala give honest service so long as they were not given
Pariyojana also include Inter-corridor & feeder adequate salaries.He therefore, enforced strict
roads, National Corridors Efficiency improvements rules against the private trade and acceptance of
and Expressways. bribes by offcials. At the same time, he persuaded
the Directors of the Company to pay handsome
20. (a)
salaries to the Company servants.
Statement 1 is correct: ASHA works as an interface
between community and the public health system. 23. (b)
Statement 2 is not correct: ASHAs, in addition Statement 1 is correct: The Permanent settlement
to other works, are selected for promoting was introduced in Bengal and Bihar in 1793,
institutional delivery in any government facility by Lord Cornwallis for the collection of land
revenue.
for both urban and rural families.

© Copyright:   www.nextias.com

https://t.me/TheHindu_Zone_official
https://t.me/IAS201819 https://t.me/PDF4Exams https://t.me/PDF4Exams

4 | CSE (Prelims) Test Series-2019


Statement 2 is not correct: Pitt’s India Act was Statement 2 is not correct: The Nawabs were
enacted in 1784 to rectify the defects of the selfish rulers absorbed in self indulgence, who
Regulating Act of 1773. It established a system of cared liitle for good administration or for the
dual control in which the commercial functions welfare of the people.Awadh had an immense
was to be controlled by the Court of Directors potential as a market for the Manchester goods,
controlled, whereas the Board of Control but couldnot be annexed under “Doctrine of
maintained its political affairs. Lapse” as the nawabs had many heirs. Hence,
Statement 3 is not correct: First newspaper “The Lord Dalhousie hit upon the idea of alleviating
Bengal Gazette” was started in 1780 by James the plight of people of Awadh and annexed
Augustus Hickey. Awadh on the pretext of misgovernnace by the
Statement 4 is correct: In 1791, Jonathan Duncan Nawabs.
started a sanskrit college at Varanasi, where he Statement 3 is correct: Annexation of Awadh
was the Resident, for study of Hindu Law and in 1856 created an atmosphere of rebellion in
philosophy. Along with the Calcutta Madarsah Awadh and in the Company’s army. Dalhousie’s
established by Warren Hasting in 1781, the action angered the Company’s sepoys, 75000 of
pupose of these institutions was to provide whom came from Awadh. This anger found the
regular supply of qualified Indians to help reflection in the Revolt of 1857.
administration of law in the courts of the British
26. (a)
Company.
Lord Curzon instituted a Police Commission
Additional Information:
in 1902 under the chairmanship of Sir Andrew
The French Revolution was a watershed event Frazer in an order to have an efficient and
in modern European history. During this disciplined police force. Curzon accepted all the
period, French citizens razed and redesigned recommendations and implemented them. He
their country’s political landscape, uprooting set up training schools for both the officers and
centuries-old institutions such as absolute the constables and introduced provincial police
monarchy and the feudal system. service.
The upheaval was caused by widespread Additional Information:
discontent with the French monarchy and the
poor economic policies of King Louis XVI, who The first Famine Commission (1878-80) was
met his death by guillotine. Although it failed to appointed under Sir Richard Strachey, it
achieve all of its goals and at times degenerated recommended to include provision of funds
into a chaotic bloodbath, it had played a critical for famine relief and construction work in the
role in shaping modern nations by showing annual budget.
the world the power inherent in the will of the 27. (a)
people. The ideas of liberty and democratic
Statement 1 is correct: The British introduced
rights were the most important legacy of the
the modern concept of the rule of law, which
French Revolution.
meant that their administration was to be carried
24. (c) out, at least in theory, in obedience to laws,
An Anglo-Indian born in 1809, Henry Vivian which clearly defined the rights, privileges and
Derozio was young and inspiring leader of the obligations of the subject and not according to
Young Bengal Movement. He taught at Hindu the personal discretion of the rulers.
College from 1826 to 1831. Derozio possesed a Statement 2 is not correct: There was an
dazzling intellect and followed the most radical exception to the principle of equality before
views of the time drawing his inspiration from the law. The Europeans and their descedants had
great French revolution. He was removed from the separate courts and even laws. In criminal cases
Hindu College in 1831 because of his radicalism they could be tried only by European judges.
and died of cholera soon at the age of 22. Ilbert Bill introduced in 1883 by C.P. Ilbert (
Law Member) during the viceroyship of Lord
25. (a) Ripon , sought to remove this racial inequality
Statement 1 is not correct: Nawabs of Awadh between Indian and European judges in courts.
had been British allies since the Battle of Buxar However, the Bill was successfully thwarted by
and had been most obedient to the British over the Europeans and Ripon had to amend the bill
the years. to satisfy the English in India and England.

© Copyright:   www.nextias.com

https://t.me/TheHindu_Zone_official
https://t.me/IAS201819 https://t.me/PDF4Exams https://t.me/PDF4Exams

Economy, Revision of Test-3 & Current Affairs_190904 | 5

28. (c) 30. (d)


Statement 1 is correct: Sir William Wedderburn The Indian army was carefully reorganised after
arrived in India in 1860 as a member of the Indian 1858, to avert the recurrence of another revolt
Civil Service. His years in rural India led him to similar to 1857.
a sympathetic understanding of the problems Statement 1 is not correct: The organisation of
of the Indian villager. In 1893, he became a Indian section of army was based on the policy
Liberal Member of Parliament and formed the of “balanced and counterpoised” or “Divide and
Indian Parliamentary Committee, which he rule”, so as to prevent the chance of uniting again
chaired from 1893 to 1900. Swami Vivekananda in an anti-British uprising. Descrimination on the
participated at the Parliament of Religions held basis of caste, region and religion was practiced
in Chicago (USA) in September 1893 and raised in recruitment to the army.
the prestige of India and Hinduism very high. Statement 2 is not correct: A fiction was created
on that the Indians consisted of “martial” and
Statement 2 is correct: The pupose of the
“non-martial classes”. Soldiers from Awadh,
committee was to voice India’s grievances in the
Bihar, Central India and South India who first
Parliament of Britain. helped the British conquer India but had later
Additional Information: taken part in 1857 Revolt, were declared to be
Wedderburn was the president of fourth session non-martial and they were no longer to be taken
(1889) and Silver jubilee session (1910) of the in army on large scale. Punjabis, Gurkhas and
Indian National Congress .An active member Pathans who helped in suppressing the revolt
of several Royal Commissions and Committees were declared to be martial and were recruited
concerned with Indian affairs, he was also in large numbers.
Chairman of the British Committee of the Statement 3 is not correct: To guarantee the
Congress from 1889 until his death in 1918. domination of Europeans, the proportion of
Europeans to Indians in the army was raised
29. (d) to one to two in Bengal army and two to five in
Statement 1 is not correct: Under the Act of 1858, Madras and Bombay army.
government was to be carried on as before by the 31. (c)
Governor General who was also given the title
Pair 1 is not correct: Following the first Anglo-
of Viceroy or Crown’s personal reepresentative. Burmese war of 1824-1826, the Treaty of Yandabo
Hence, there was no separate post of the Viceroy was signed in 1826. As per the treaty, the
was created by abolishing the post of Governor government of Burma agreed to pay one crore
General of India. Lord Canning had the unique rupees as war compensation and to abandon
opportunity to become the Governor-General as all claims to Assam, Cachar and Jaintia. By this
well as the first Viceroy. treaty the British deprived Burma of most of its
Statement 2 is not correct: In 1876, Queen coastline, and acquired a firm base in Burma for
Victoria assumed the title of the Empress of India future expansion.
to emphasize British sovereignty over the entire Pair 2 is correct: After the second Afghan war
Indian subcontinent. (1873-79), the Treaty of Gandmark was signed
Additional Information: by Yakub Khan. By this treaty British secured
certain border districts, the right to keep a
Main provisions of 1858 Act: Resident at Kabul and control over Afghanistan
(i) East India Company’s rule came to an end foreign policy.
and the Indian administration came under Pair 3 is correct: Treaty of Salbai was signed
the direct control of the Crown. following the first Anglo- Maratha war of 1775-
(ii) In England, the Court of Directors and 82. The war didnot end in victory for either side,
Board of Control were abolished. In their but it did give the British 20 years of peace with
place came the Secretary of State for India the Marathas, the strongest Indian power of
and India Council were established. The the day. The treaty enabled the British to exert
Secretary of State would be a member of the pressure on Mysore, as the Marathas promised to
British cabinet. Sir Charles Wood was made help them in recovering territories from Haider
the first Secretary of State for India. Ali. In short, the treaty saved the British from the
(iii) All the previous treaties were accepted and combined opposition of Indian powers (Marathas,
honoured by the Act. Mysore and Hyderabad) and they utilized this to
consolidate their rule over Bengal Presidency.

© Copyright:   www.nextias.com

https://t.me/TheHindu_Zone_official
https://t.me/IAS201819 https://t.me/PDF4Exams https://t.me/PDF4Exams

6 | CSE (Prelims) Test Series-2019


32. (d) 34. (d)
The war of 1742 in Europe between France and Statement 1 is not correct: It was through the
England soon spread to India where the two East Charter Act of 1813, for the first time British took
India Companies clashed against each other. the responsibility for education of native Indians
Though war in Europe ended in 1748, the rivalry and a sum of one lakh rupees was to be set aside
in trade and over possessions in India continued. for the same.
Statement 1 is not correct: Dupleix, the French Statement 2 is correct: Through the Act the island
Governor-General had evolved the strategy of Saint Helena was vested in His Majesty, hence
the Charter Act of 1833 was also known as Saint
of using the well-disciplined French Army to
Helena Act. It provided for addition of a Law
intervene in the internal quarrels of the Indian
Member to the Governor-General’s Council. T.
native rulers and by supporting against the other, B. Macaulay became the first Law Member of the
securing monetary, commercial or territorial Governor- General-in-Council.
favours from the victor. He planned to use the
Statement 3 is not correct: The Charter Act of
resources and armies of the local rajas, nawabs
1833 provided that the administration should
and chiefs to serve the interest of the French take steps to ameliorate the conditions of slaves
company and to expel the English from India. and to ultimately abolish slavery. Slavery was
Statement 2 is not correct: The battle of abolish which was ultimated abolished in 1843.
Wandiwash was fought in 1760 between the Additional Information:
British and French armies. The English won Other important provisions of 1833 Act:
the battle under the leadership of General Eyre
(i) The English East India Company ceased to
Coot. Within a year, the French had lost all their
be a commercial agency in India. In other
possessions in India. The war ended in 1763 with words, it would function hereafter as the
the signing of the Treaty of Paris. The French political agent for the Crown.
factories in India were resorted but they could
(ii) The Company’s monopoly over trade with
no longer be fortified. They could serve only as
China and in tea also ended.
a centre of trade, and now the French lived in
(iii) The Governor-General of Fort William was
India under British protection.
hereafter called ‘the Governor- General
33. (c) of India’. Thus, Bentinck was the first
The correct sequence of the establishments Governor-General of India’.
from north to south on India’s map is 1-2-3, as (iv) The Act categorically stated ‘that no native
explained below: of India, nor any natural born subject of His
Majesty, should be disabled from holding
(i) First cotton textile mill of India was started
any place, office, or employment, by reason
in Bombay by Cowasjee Nanabhoy in 1853. of his religion, place of birth, descent or
(ii) The English opened their first Factory at colour”. It was this enactment which laid the
Masulipatnam in 1611.The English East foundation for the Indianisation of public
India Company was established on 31 services.
December 1600 as per the Royal Charter
35. (b)
issued by the Queen of England, Elizabeth I.
The Company had sent Captain Hawkins to Statement 1 is not correct: Warren Hastings
adopted the “Policy of Ring fence” in wars against
the court of the Mughal Emperor, Jahangir
the Marathas and Mysore with an aim to create
in 1608 to secure permission to establish a
buffer zones to defend the Company’s frontier.
“factory” (store house of goods) at Surat.
It was turned down initially. However, in Statement 2 is correct: To achieve political aims
of British, Lord Wellesley (1798-1805) relied
1613, Jahangir issued the firman permitting
on three methods: the system of “Subsidiary
the East India Company to establish its first
alliance”, outright war and the assumption of
trading post at Surat.
territories of previously subordinated rulers.
(iii)
The French East India Company was Under the Subsidiary Alliance system, the ruler
founded in 1664. It was firmly established of allying Indian state was compelled to accept
at Chandernagore near Calcutta and permanent stationing of a British force within his
Pondicherry on the east Coast. The factory territory and to pay a subsidy for its maintenance.
at Pondicherry was fully fotified. In return, British undertook to defend the ruler

© Copyright:   www.nextias.com

https://t.me/TheHindu_Zone_official
https://t.me/IAS201819 https://t.me/PDF4Exams https://t.me/PDF4Exams

Economy, Revision of Test-3 & Current Affairs_190904 | 7

from his enemies. Hyderabad was first to sign identified as Brahamin, kshatriya, vaishya or
Subsidiary Treaties with Lord Wellesley in 1798 shudra according to the occupation the person
and 1800. Nawab of Awadh was forced to sign followed. He was opposed to idolatry, ritual and
the subsidiary treaty in 1801. priesthood, and particularly to the prevalent
Statement 3 is correct: Lord Canning proclaimed caste practices and popular Hinduism preached
the new Government at Allahabad on 1 by brahmins.
November 1858 in accordance with the Queen’s Statement 3 is not correct: Arya Samaj emphasized
Proclamation. The latter has been called the the importance of western education, which
Magna Carta of the Indian people; it disclaimed was reflected in the establishment of Dayanand
any extension of territory, promised religious Anglo vedic schools (first at Lahore in 1886).
toleration, guaranteed the rights of Indian
38. (a)
princes and pledged equal treatment to her
subjects, Indians and Europeans. Statement 1 is corect: Many different kinds of
movements influenced each other in North India,
36. (b) and people from different religions and castes
Statement 1 is not correct: In 1800, Wellesley thronged to holy places to listen to preachers of
has set up the Fort William College for training new religions. People such as Kabir and Guru
of new recruits. However, it was disapproved Nanak rejected all orthodox religions but people
by the Court of Directors in 1806 and instead such as Tulsidas and Surdas accepted existing
the East India College was set up at Haileybury beliefs and made them accessible to all.
in England to impart two years’ training to the Statement 2 is not correct: Guru Nanak’s
recruits. successor, Guru Angad compiled the
Statement 2 is not correct: It was Lee compositions of Guru Nanak to which he added
Commission in its report in 1924, recommended his in a new script known as Gurmukhi.
direct recruitment to ICS on basis of 50:50 Statement 3 is correct: The three successors
pairity between the Europeans and Indians to be of Guru Angad also wrote under the name
achieved in 15 years. of “Nanak” and all of their compositions
Statement 3 is correct: INC was demanding were compiled by Guru Arjan in 1604. To this
for lowering the age limit for recruitment and compilation were added the writings of other
simultaneous examination in India and England. figures like Shaikh Farid, Sant Kabir, Bhagat
The House of Commons in England in 1893, Namdev and Guru Tegh Bahadur. In 1706 this
passed a resolution supporting holdings of compilation was authenticated by Guru Tegh
simultaneous examination in India and England, Bahadur’s son and successor, Guru Gobind
but the resolution was never implemented. Singh. It is now known as Guru Granth Sahib,
Additional Information: the holy scripture of the Sikhs.
(i) Satyendra Nath Tagore became the first 39. (d)
Indian to qualify for the Indian Civil Service Statement 1 is not correct: Young Bengal
in 1863. movement was a radical trend arose among the
(ii) Aitchison Committee on public Service bengali Intellectuals during the late 1820s to
(1886) recommended for rasing the age limit 1830s. This trend was more modern than even
to 23 and classification of the civil services Rammohan Roy’s. The movement was led by the
into Imperial ICS (Exam in England), young Anglo- Indian Henry Vivian Derozio.The
provincial Civil Service (exam in India) and Derozians attacked old and decadant customes,
Subordinate Civil service (exam in India) rites and traditions, and advocated women’s
37. (a) rights. They carried forward the tradition of
Rammohan’s tradition of educating the people in
Statement 1 is not correct: The first formal unit
social, political and economic questions through
of Arya Samaj was set up by Dayanand Saraswati
newspapers, pamphlets and public associations.
at Bombay in 1875 and later the headquarters of
Surendranath Banerjee described the Derozians
Samaj were establsihed at Lahore.
as “ the pioneers of the modern civilization of
Statement 2 is correct: Dayanand gave the Bengal”.
slogan “Back to Vedas”. He subscribed to the
Statement 2 is not correct: The movement was
vedic notion of chaturvarna system in which
inspired from the ideas of liberty and equality of
a person was not born in any caste but was
the great French Revolution.

© Copyright:   www.nextias.com

https://t.me/TheHindu_Zone_official
https://t.me/IAS201819 https://t.me/PDF4Exams https://t.me/PDF4Exams

8 | CSE (Prelims) Test Series-2019


Additional Information: Statement 1 is correct: Horizon 2020 is the
The modern idea of natural rights grew out of the largest multinational programme dedicated
ancient and medieval doctrines of natural law, to research and innovation.This means that
i.e., the belief that people, as creatures of nature researchers, universities, research organisations,
and God, should live their lives and organize companies and non-governmental organisations
their society on the basis of rules and precepts from across the globe can apply to participate in
laid down by nature or God. The Declaration the activities of the Work Programme carried out
of Independence of the United States lists life, mainly through calls for proposals.
liberty, and the pursuit of happiness as natural Statement 2 is not correct: It is open to the whole
rights. world and not restricted only to developing
countries.
40. (a)
The Lottery Committee of 1817 was associated 43. (a)
with the town planning of Kolkata. It was so India’s logistics sector is highly defragmented
called because funds for city development were and the aim is to reduce the logistics cost from
raised through public lotteries. the present 14% of GDP to less than 10% by 2022.
Statement 1 is correct: The National logistic
41. (a)
portal will link all the stakeholders of EXIM,
The National Commission on Backward domestic trade and movement and all trade
Classes(NCBC) is a body set up under the activities on a single platform. The portal will
National Commission for Backward Classes Act, be implemented in phases and will fulfil the
1993. It has the power to examine complaints commitment of the Government of India to
regarding inclusion or exclusion of groups enhance trade competitiveness, create jobs, boost
within the list of backward classes, and advise India’s performance in global rankings and pave
the central government in this regard. the way for India to become a logistics hub.
Statement 1 is correct: It seeks to grant the Statement 2 is not correct: A National Logistics
National Commission on Backward Classes Portal is being developed to ensure ease of trading
(NCBC) constitutional status, at par with the in the international and domestic markets.
National Commission for Scheduled Castes
Statement 3 is not correct: A National Logistics
(NCSC) and the National Commission for
Portal is being developed by the Ministry
Scheduled Tribes.
of Commerce and Industry. Department of
Statement 2 is not correct: Currently, under Commerce will create a portal which will be a
the Constitution the National Commission for single window online market place for trade and
Scheduled Castes (NCSC) has the power to look will connect business, create opportunities and
into complaints and welfare measures with bring together various ministries, departments
regard to Scheduled Castes, backward classes
and the private sector. Stakeholders like traders,
and Anglo-Indians. The Bill seeks to remove the
manufacturers, logistics service providers,
power of the NCSC to examine matters related to
infrastructure providers, financial services,
backward classes only. The Bill seeks to establish
Government departments and groups and
the NCBC under the Constitution, and provide it
associations will all be on one platform.
the authority to examine complaints and welfare
measures regarding socially and educationally 44. (c)
backward classes. A Geographical Indication or a GI is an
Statement 3 is correct: Under the Constitution indication used on products that have a specific
Amendment Bill, the NCBC will comprise of geographical origin and possess qualities or
five members appointed by the President. Their a reputation that are due to that origin. Such
tenure and conditions of service will also be a name conveys an assurance of quality and
decided by the President through rules. distinctiveness which is essentially attributable
42. (a) to its origin in that defined geographical locality.
The Work Programme 2018-2020 of the EU's Option 1 is correct: Alphonso from Ratnagiri,
Research and Innovation funding programme Sindhudurg, Palghar, Thane and Raigad districts
Horizon 2020 was launched by the European of Maharashtra, is registered as Geographical
Commission on 27/10/2017. Indication (GI).

© Copyright:   www.nextias.com

https://t.me/TheHindu_Zone_official
https://t.me/IAS201819 https://t.me/PDF4Exams https://t.me/PDF4Exams

Economy, Revision of Test-3 & Current Affairs_190904 | 9

Option 2 is correct: Bihar's Shahi litchi has semen and oocytes from wildlife and successfully
got the Geographical Indication (GI) and has reproducing endangered blackbuck, spotted
become an exclusive brand in the national and deer and Nicobar pigeons. Through this work,
international market. The famous Shahi litchi, it has established Genetic Resource Bank for
which is famous for its sweet, juicy, mostly Indian wildlife.
grown in Muzzaffarpur and neighbouring Statement 2 is correct: It would facilitate
districts including East champaran. exchange of genetic material between the
Option 3 is not correct: Madhya Pradesh has lost Indian zoos for maintaining genetic diversity
and conservation management made accessible
its claim to market rice under the basmati tag,
to scientists and wildlife managers for
with the Geographical Indications (GI) Registry
implementing conservation programs.
issuing an order excluding the state from the
area officially demarcated for cultivation of this 47. (d)
aromatic paddy variety. 3D printing or additive manufacturing is a
Option 4 is correct: The famous Kadaknath process of making three dimensional solid objects
chicken meat from Jhabua district of Madhya from a digital file. The creation of a 3D printed
Pradesh has got a Geographical Indication (GI) object is achieved using additive processes. In
tag. an additive process an object is created by laying
down successive layers of material until the
45. (d) object is created. Each of these layers can be seen
India began the development of the ballistic as a thinly sliced horizontal cross-section of the
missile defence system in 1998. The development eventual object.
was planned in two stages. The first phase was Statement 1 is correct: Coral reefs are being
challenging due to the complex technologies and threatened by climate change, which is causing
indigenous mission systems employed in the coral bleaching at an alarmingly high rate.
mission. This poses innumerable problems for these
Statement 1 is not correct: The Indian Defence underwater ecosystems, but countries are
Research and Development Organisation turning to 3D printing to help try and save these
(DRDO) is developing a two-tier Ballistic Missile critical habitats. To help promote the growth of a
Defence (BMD) system that provides a multi- coral reef ecosystem in the Maldives, a team has
layered shield against ballistic missile attacks. designed and 3D printed an artificial reef made
Statement 2 and 3 are not correct: The Ballistic from hundreds of ceramic and concrete modules.
Missile Defence (BMD) System it is a double- The 3D printed reef is being used by the Summer
tiered system consisting of two land and sea- Island Maldives resort to build a new coral reef
based interceptor missiles, namely the Prithvi ecosystem.
Air Defence (PAD) missile for high altitude Statement 2 is correct: New research published
interception, and the Advanced Air Defence in ACS Applied Energy Materials shows that it's
(AAD) Missile for lower altitude interception. possible to 3D-print lithium-ion batteries into
The two-tiered shield should be able to intercept whatever shape you need. In order to make the
any incoming missile launched 5,000 kilometres batteries conductive, the team led by Christopher
away. The system also includes an overlapping
Reyes and Benjamin Wiley infused the polylactic
network of early warning and tracking radars, as
acid (PLA) usually used in 3D printing with
well as command and control posts. Hence,
an electrolyte solution. The researchers also
46. (d) incorporated graphene and carbon nanotubes
Effective conservation measures include both into the design of the case to help increase
in situ habitat preservation, species protection conductivity. After these design modifications,
and ex situ conservation (captive breeding in the team was able to 3D print an LED bracelet,
controlled environment to restock original wild complete with a custom-sized lithium-ion
populations). In order to support both these battery.
measures using biotechnological tools and Statement 3 is correct: Scientists have developed
techniques in an innovative manner, LaCONES a method to 3-D print cells to produce human
was established. tissue such as ligaments and tendons to greatly
Statement 1 is correct: CCMB-LaCONES is the improve a patient's recovery. A person with a
only laboratory in India that has developed badly damaged ligament, tendon, or ruptured
methods for collection and cryopreservation of

© Copyright:   www.nextias.com

https://t.me/TheHindu_Zone_official
https://t.me/IAS201819 https://t.me/PDF4Exams https://t.me/PDF4Exams

10 | CSE (Prelims) Test Series-2019


disc could simply have new replacement Coordinating Council (ICC) of Man and Biosphere
tissue printed and ultimately implanted in the (MAB) Programme of UNESCO held at Palembang,
damaged area. Indonesia, from July 23-27, 2018.

48. (c) 50. (c)


Indian scientist-academician, N Raghuram, has Asia-Pacific Institute for Broadcasting
been elected Chair of the International Nitrogen Development (AIBD), established in 1977 under
Initiative (INI), a global policy making initiative. the auspices of UNESCO. The AIBD is mandated
The International Nitrogen Initiative (INI) is to achieve a vibrant and cohesive electronic media
an international program, set up in 2003 under environment in the Asia-Pacific region through
sponsorship of the Scientific Committee on policy and resource development
Problems of the Environment (SCOPE) and from Statement 1 is correct: It is a unique regional inter-
the International Geosphere-Biosphere Program governmental organisation servicing countries
(IGBP).The INI holds a conference every three of the United Nations Economic and Social
years, inviting members of the international Commission for Asia and the Pacific (UN-ESCAP)
nitrogen community to meet up and discuss in the field of electronic media development.
ideas and exchange knowledge on nitrogen Statement 2 is correct: India has been elected as the
issues. president of Asia-Pacific Institute for Broadcasting
Development (AIBD) for the first time
Statement 1 is correct: One of the key aims of
the INI is to optimize nitrogen’s beneficial role in 51. (c)
sustainable food production. The Anglo-Nepalese War (1814–16) ended with the
Statement 2 is correct: It also aims to minimize signing of the Treaty of Sagauli in 1816.
nitrogen’s negative effects on human health and Statement 1 is correct: The British Empire in India
the environment resulting from food and energy now reached the Himalayas which gave them greater
production. facilities for trade with Central Asia.
49. (a) Statement 2 is correct: The Gurkhas gave added
strength to the British-Indian army by joining it in
Khangchendzonga National Park is located at the
large numbers.
heart of the Himalayan range in northern India
(State of Sikkim) and it includes a unique diversity Additional Information
of plains, valleys, lakes, glaciers and spectacular, By signing this treaty Nepal accepted a British
snow-capped mountains covered with ancient Resident. It ceded the districts of Garhwal and
forests, including the world’s third highest peak, Kumaon and abandoned claims to the Tarai areas.
Mount Khangchendzonga. It also withdrew from Sikkim.
Statement 1 is correct: The Khangchendzonga 52. (a)
National Park (KNP) lies within the Himalaya The Indian Government, Maharaja Ranjit Singh,
global biodiversity hotspot and displays an and Shah Shuja signed a treaty at Lahore on 26
unsurpassed range of sub-tropical to alpine June 1838.
ecosystems. The Himalayas are narrowest here, Lord Auckland was the Governor General at that
resulting in extremely steep terrain, which time.
magnifies the distinction between the various eco-
Additional Information:
zones.
By this treaty Maharaja Ranjit Singh and the Indian
Statement 2 is correct: Khangchendzonga National
Govt promised to help Shah Shuja capture power
Park has been inscribed as India’s first “Mixed
in Afghanistan by replacing Dost Muhammed
World Heritage Site” on UNESCO World Heritage
and, in return, Shah Shuja promised not to enter
List, by fulfilling the nomination criteria under
into negotiations with any foreign state without the
both natural and cultural heritage. consent of the British and the Punjab Government.
Statement 3 is not correct: The Khangchendzonga
Biosphere Reserve has become the 11th Biosphere 53. (d)
Reserve from India that has been included in the British took many measures to prevent Afghanistan’s
UNESCO designated World Network of Biosphere inclination towards Russia.
Reserves (WNBR). The decision to include Action 1 is correct: After 1864 the policy of non-
Khangchendzonga Biosphere Reserve in WNBR interference was vigorously pursued by Lord
was taken at the 30th Session of International Lawrence and his two successors.

© Copyright:   www.nextias.com

https://t.me/TheHindu_Zone_official
https://t.me/IAS201819 https://t.me/PDF4Exams https://t.me/PDF4Exams

Economy, Revision of Test-3 & Current Affairs_190904 | 11

Action 2 is correct: As Russia again turned its 2. The Arms Act of 1878, which disarmed the
attention to Central Asia after its defeat in the Crimean people, appeared to them as an effort to
War, the British followed the policy of strengthening emasculate the entire nation.
Afghanistan as a powerful buffer. They gave the Amir 3. The Vernacular Press Act of 1878
of Kabul aid and assistance to help him discipline his
rivals internally and maintain his independence from 56. (d)
foreign enemies. Thus, by a policy of non-interference Statement 1 is not correct: It was the Charter
and occassional help, the Amir was prevented from Act of 1833 by which debts of the Company
aligning himself with Russia. were taken over by the Government of India
Action 3 is correct: The policy of non-interference which was also to pay its shareholders a
did not, however, last very long. From 1870 onwards dividend on their capital.
there was a resurgence of imperialism all over the
Statement 2 is not correct: It was the Charter
world. British officials and public opinion were again
Act of 1813 allowed Christian missionaries to
haunted by the hysterical fear of a Russian invasion
enter India without restrictions.
of India, the 'brightest jewel' in the British Empire.
To force British terms on the Amir a new attack on 57. (a)
Afghanistan was launched in 1878. The Indian Currency Committee or Fowler
54. (b) Committee was a government committee
appointed by the British Government of India on
Statement 1 is not correct: Lord Dufferin was the
29 April 1898 to examine the currency situation
Viceroy of British India when INC was formed.
in India. The committee recommended that the
Statement 2 is correct: George Yule was the first official Indian rupee be based on the gold standard
British President of INC and presided over the and the official exchange rate of the rupee be
Allahabad session in 1888. established at 15 rupees per British sovereign, or 1
Statement 3 is correct: Maulana Abul Kalam shilling and 4 pence per rupee.
Azad presided over the 1940 session in
Ramgarh. He remained the president of INC 58. (c)
till 1946. Statement 1 is not correct: Treaty of Yandabo was
signed in February 1826 which ended the First
55. (b) Anglo-Burmese War.
Statement 1 is correct: During Lytton’s Statement 2 is correct: The Burmese occupation
viceroyalty from 1876-80 most of the import of Arakan and Assam led to continuous friction
duties on British textile imports were removed along the ill-defined border between Bengal
to please the textile manufacturers of Britain. It and Burma. Burmese occupation of Manipur
created a wave of anger in the country and led and Assam provided another source of conflict
to widespread nationalist agitation. between the two. It was looked upon by the British
Statement 2 is not correct: It was passed authorities as a serious threat to their position in
during the reign of Lord Curzon India. For several decades, British had been trying
Statement 3 is correct: The holding of the to persuade the Government of Burma to sign
Imperial Durbar at Delhi in 1877 at a time a commercial treaty with them and to exclude
when the country was suffering from a terrible French traders from Burma. The British seized this
famine led people to believe that their rulers opportunity to declare war on Burma.
cared very little for their lives. Second Anglo-Burmese War: British timber firms
Statement 4 is correct: In 1878, the Government had begun to take interest in the timber resources
announced new regulations reducing the of Upper Burma. Moreover, the large population of
maximum age limit for sitting in the Indian Burma appeared to the British to be a vast market
Civil Service Examination from 21 years to for the sale of British cotton goods and other
19. The new regulations further reduced their manufactures. The British, already in occupation
chances of entering the Civil Service. of Burma's two coastal provinces, now wanted to
Additional Information establish commercial relations with the rest of the
country, but, the Burmese Government would not
Other steps takes during Lytton’s regime
permit further foreign commercial penetration.
which arose discontent among the Indians:
British merchants began to complain of “lack of
1. The Second War against Afghanistan facilities for trade” and of “oppressive treatment"
aroused agitation against the heavy cost of by the Burmese authorities at Rangoon.
this imperialist war.

© Copyright:   www.nextias.com

https://t.me/TheHindu_Zone_official
https://t.me/IAS201819 https://t.me/PDF4Exams https://t.me/PDF4Exams

12 | CSE (Prelims) Test Series-2019


Statement 3 is correct: After the First World War, 61. (d)
a vigorous modern nationalist movement arose Statement 1 is not correct: According to recent
in Burma. A wide campaign of boycotting British annual report by RBI, Mauritius remained the top
goods and administration was organised and source of foreign direct investment (FDI) into India
the demand for Home Rule was put forward. in 2017-18 followed by Singapore. India received
The Burmese nationalists soon joined hands USD 37.3 billion capital inflow in 2017-18 as
with the Indian National Congress. In 1935 the compared to USD 36.3 billion in the previous fiscal.
British separated Burma from India in the hope of
Statement 2 is not correct: According to the
weakening the Burmese struggle for freedom. The
UNCTAD's Investment Trends Monitor (2018),
Burmese nationalists opposed this step
India was the 10th largest recipient of global FDI
59. (a) in 2017 and remained the topmost destination
Statement 1 is correct: After the mutiny of 1857, for greenfield capital investment – even ahead of
the Peel Commission was appointed to look into China and the US, if reckoned on an approval basis
the military affairs of India. It recommended that (FDI market intelligence 2017).
"the native army should be composed of different
62. (b)
nationalities and castes, and as a general rule,
mixed promiscuously through each regiment.” National Health Protection Scheme also known
Therefore, during the next few years regiments as Ayushman Bharat, will subsume the on-
which had mutinied were disbanded, castes going centrally sponsored schemes - Rashtriya
were more evenly mixed across the regiments, Swasthya Bima Yojana (RSBY) and the Senior
recruitment remained focused on Punjab which Citizen Health Insurance Scheme (SCHIS).
remained loyal during the mutiny, and the regional Pradhan Mantri Jeevan Jyoti Yojana: It is an
elements like the Punjab, Hindustan, Bombay and insurance scheme available to people between 18
Madras, were carefully kept separate. and 50 years of age with bank accounts. It has
Statement 2 is not correct: The Indian states an annual premium of ₹330. The cover under
committee which was formed in 1928 under Sir PMJJBY is for death only and hence benefit will
Harcourt Butler to investigate and clarify the accrue only to the nominee. It is not the part of
relationship between the paramount power and National Health Protection Scheme.
the Princes scarcely provided in its Report (1929)
63. (b)
any solace for the beleaguered princes. It gave
them a concession in the form of a promise that Pakistan, Myanmar, Maldives are not part of
paramountcy would not be transferred without IORA.
their consent to any democratically elected Additional information:
government in British India; but at the same time, The purpose of Indian Ocean Rim Association
it reaffirmed the supremacy of paramountcy with is promoting sustained growth and balanced
unlimited power-even to suggest constitutional development within the Indian Ocean region,
changes in a particular state if there was widespread IORA strengthens cooperation and dialogue
demand for such reforms. with Member States namely:
60. (b) Australia, Bangladesh, Union of Comoros, India,
Indonesia, Iran, Kenya, Madagascar, Malaysia,
Statement 1 is not correct: It was signed during
Mauritius,Mozambique, Oman, Seychelles,
the second Anglo-Afghan war when Lord Lytton
Singapore, Somalia , South Africa, Sri Lanka,
was the Viceroy of India. Lord Auckland was the
Tanzania, Thailand, United Arab Emirates and
Governor General during the first Anglo-Afghan
Yemen.
war.
Statement 2 is correct: By this treaty British gained 64. (b)
control over Afghanistan's foreign policy. Statement 1 is not correct: Financial Inclusion
Additional Information Index will be launched by Department of
To force their terms on the Amir, British launched Financial Services (DFS), Ministry of Finance not
a new attack on Afghanistan in 1878. Peace came in Ministry of Commerce and industry.
May 1879 when Sher Ali‟s son, Yakub Khan, signed Statement 2 is correct: It will measure a basket
the Treaty of Gandamak by which the British secured of formal financial products and services that
all they had desired. They secured certain border includes savings, remittances, credit, insurance
districts and the right to keep a Resident at Kabul. and pension products.

© Copyright:   www.nextias.com

https://t.me/TheHindu_Zone_official
https://t.me/IAS201819 https://t.me/PDF4Exams https://t.me/PDF4Exams

Economy, Revision of Test-3 & Current Affairs_190904 | 13

Statement 3 is correct: The index will have three Statement 2 is not correct: According to a press
measurement dimensions; (i) Access to financial release issued by WWF, Nepal would become
services (ii) Usage of financial services and (iii) the first country to double its national tiger
Quality. So this statement is correct. population since the launch of ambitious TX2
goal.
65. (c)
Additional Information:
Statement 1 is not correct: SDRF is located in the
‘Public Account’ under ‘Reserve Fund’. This program was started in 2010 at St. Pittsburg
Summit. The Tiger Summit resulted in the Global
Statement 2 is not correct: The government
Tiger Recovery Plan which outlines how each
of India has taken a decision to enhance its
country can reach the Tx2 target.
contribution in the State Disaster Relief Fund
fron 75% to 90% for all states of India. Hence this 69. (a)
statement is not correct. Statement 1 is correct: The NOTA option was
Statement 3 is correct: The financial assistance first used in the 2013 assembly elections held in
from SDRF/NDRF is for providing immediate four states- Chhattisgarh, Mizoram, Rajasthan and
relief and is not compensation for loss/ Madhya Pradesh and the Union Territory, Delhi.
damage to properties /crops. Further, the
Statement 2 is not correct: NOTA option does
provision for disaster preparedness, restoration,
not mean right to reject and won't affect election
reconstruction and mitigation are not a part of
results. For example even if there are 99 NOTA
SDRF. The Disaster Management Act specifies
that for such activities a separate fund called votes out of 100, and candidate X gets just one vote,
Disaster Mitigation Fund has to be constituted. X is the winner.
Statement 3 is not correct: The Supreme Court
66. (d) recently ruled that the None of The Above (NOTA)
Statement 1 is not correct: The standards are set option cannot be made available in Rajya Sabha
by the Directorate of Marketing and Inspection elections. It was observed that the NOTA option is
an attached Office of the Department of applicable only for direct elections and not indirect
Agriculture, Cooperation and Farmers Welfare elections such as the Rajya Sabha polls.
under Ministry of Agriculture & Farmers
So, Both statement 2 and 3 are not correct.
Welfare, not by FSSAI.
Statement 2 is not correct: The present AGMARK 70. (d)
standards cover quality guidelines for 222 Giving a major boost to the pro-farmer initiatives,
different commodities spanning a variety of the Union Cabinet has approved a new Umbrella
pulses, cereals, essential oils, vegetable oils, fruits Scheme “Pradhan Mantri Annadata Aay
and vegetables and semi-processed products like SanraksHan Abhiyan’ (PM-AASHA). The Scheme
vermicelli. is aimed at ensuring remunerative prices to the
Additional Information: farmers for their produce as announced in the
AGMARK is legally enforced in India by the Union Budget for 2018.
Agricultural Produce (Grading and Marking) It has following components:
Act of 1937 (and amended in 1986). 1. Price Support Scheme (PSS),
67. (b) 2. Price Deficiency Payment Scheme (PDPS)
Statement 1 is correct: ASTRA is an all weather 3. Pilot of Private Procurement & Stockist Scheme
beyond-visual-range air-to-air missile. (PPPS).
Statement 2 is not correct: ASTRA is an
71. (a)
indigenous project of DRDO.
Statement 3 is correct: The Astra has an officially Statement 1 is not correct: National Nutrition
stated range of 75 kilometers. Mission is a flagship programme of the Ministry
of Women and Child Development (MWCD), not
68. (a) Ministry of Health and Family Welfare.
Statement1 is correct: World Wildlife Statement 2 is correct: All the States and districts
Foundation’s (WWF) ‘Tx2’ programme set an will be covered in a phased manner i.e. 315 districts
ambitious and visionary species conservation in 2017-18, 235 districts in 2018-19 and remaining
goal as the governments of the all tiger range districts in 2019-20. NNM targets to reduce stunting
countries pledged to double the number of wild by 2% per annum. Although the target to reduce
tigers by 2022, not all tiger reserve countries. Stunting is atleast 2% p.a., Mission would strive to

© Copyright:   www.nextias.com

https://t.me/TheHindu_Zone_official
https://t.me/IAS201819 https://t.me/PDF4Exams https://t.me/PDF4Exams

14 | CSE (Prelims) Test Series-2019


achieve reduction in Stunting from 38.4% (NFHS- 74. (b)
4) to 25% by 2022 (Mission 25 by 2022). Statement 1 is not correct: The Central Pollution
Statement 3 is correct: National Nutrition Mission Control Board (CPCB), statutory organisation, was
aims to achieving its targets by utilising the constituted in September, 1974 under the Water
Anganwadi services. It also aims to improve the (Prevention and Control of Pollution) Act, 1974.
quality of Anganwadi Service delivery for targeted Statement 2 is correct: The parliament enacted
malnutrition eradication. the Water (Prevention and Control of Pollution)
Act, 1974 with a view to maintaining and restoring
72. (c)
wholesomeness of our water bodies. One of
Statement 1 is not correct: COMCASA, a military- the mandates of CPCB is to collect, collate and
information sharing pact, which will give India disseminate technical and statistical data relating to
access to the US technology to keep an eye on water pollution.
China and the neighbourhood, was signed at the Additional Information:
2+2 bilateral summit. India had signed Logistics
Other functions of CPCB:
Exchange Memorandum of Agreement (LEMOA)
1. Advise the Central Government on any matter
– in 2016.
concerning prevention and control of water and
Statement 2 is correct: The four foundational air pollution and improvement of the quality of
agreements are – Logistics Exchange Memorandum air.
of Agreement (LEMOA), COMCASA, General
2. Plan and cause to be executed a nation-wide
Security Of Military Information Agreement
programm for the prevention, control or
(GSOMIA)(Signed in 2002) and Basic Exchange
abatement of water and air pollution;
and Cooperation Agreement for Geo-spatial
Cooperation (BECA). Only BECA is yet to be 75. (d)
signed. Statement 1 is correct: Akash misslies are
Statement 3 is correct: The Communications indigenously developed short-range surface-to-air
Compatibility and Security Agreement (Comcasa) missile (SRSAM) system.
will give India access to advanced new defence Statement 2 is correct: The Defence Research and
systems like armed drones and will enable Development Organisation (DRDO) developed
the armed forces to exploit existing US-origin Akash as part of the Integrated Guided Missile
platforms much more efficiently. Signing the Development Programme initiated in 1984.
CISMOA would enable India to get encrypted Statement 3 is correct: Akash has a range of 25
communications equipment and systems allowing km and can engage multiple targets at a time in
military commanders to communicate with aircraft all-weather conditions. It can employ multiple air
and ships through a secure network. targets while operating in fully autonomous mode.

73. (a) 76. (d)


Statement 1 is correct: ICC is the world’s first Statement 1 is not correct: Supreme court declared
permanent international criminal court. The Rome Aadhaar a “document of empowerment” and an
Statute of the International Criminal Court (often “unparalleled” identity proof. So Supreme Court
referred to as the International Criminal Court explicitly allowed statutory examining bodies to
use Aadhar as a proof of identity.
Statute or the Rome Statute) is the treaty that
established the International Criminal Court (ICC). Statement 2 is correct: It was held that permission
of parents and guardians was a must before
Statement 2 is not correct: India is not a founding enrolling children into Aadhaar. Children once
member of ICC. Infact India has not even signed they attained the age of majority could opt out of
the Rome Statute. Aadhaar, because as adults they have the right to
Statement 3 is not correct: The International decide for themselves.
Criminal Court (ICC) investigates and, where Statement 3 is correct: The Court held that
warranted, tries individuals charged with the authentication records should not be retained for
gravest crimes of concern to the international more than six months. It declared the archiving
community: genocide, war crimes, crimes against of records for five years as “bad in law.” It
humanity and the crime of aggression. also prohibited the creation of a metabase for
transactions.

© Copyright:   www.nextias.com

https://t.me/TheHindu_Zone_official
https://t.me/IAS201819 https://t.me/PDF4Exams https://t.me/PDF4Exams

Economy, Revision of Test-3 & Current Affairs_190904 | 15

Additional Information: banking services. So the statement is true.


Other observations of the Supreme Court: Statement 2 is correct: one of the service of
1. The statute only sought “minimal” biometric India Post Payment Bank (IPPB) includes
information, and this did not amount to "person to person remittances both domestic
invasion of privacy. and cross border." Under it special focus
2. Upholding the passage of the Aadhaar Act will be on providing economical, safe and
as a Money Bill, the Supreme Court said convenient money transfer facilities to
neither were individuals profiled nor their migrant labourers, NRIs remitting money to
movements traced when Aadhaar was used relatives, institutions etc.
to avail government benefits under Section 7 Statement 3 is not correct: IPPB comes
of the Aadhaar Act of 2016.
under the category of "payment bank".
3. The court insulated children from the Since liquidity is the most important aspect
Aadhaar regime. The card was not necessary required for such banks they will be bound
for children aged between six and 14 under the
by the reserve requirement rules of RBI So
Sarva Shiksha Abhiyan as right to education
It is not exempted from meeting Statutory
was a fundamental right.
Liquidity Ratio requiremenst of RBI.
4. The court directed the government and the
Unique Identification Authority of India Additional Information:
(UIDAI) to bring in regulations to prevent Payments banks can collect deposits of up to
rightfully entitled people from being denied Rs.1 lakh, provide payments and remittance
benefits. services and distribute third-party financial
products. They won’t be able to give loans and
77. (c)
issue credit cards, but can provide debit cards
Guiding concept: Relation between value and Internet banking services. Essentially,
of rupee and its demand: As the demand
they will mobilize deposits on behalf of other
of rupee against dollar increases, rupee
banks, acting as a business correspondent.
stabilises.
Small banks, on the other hand, will offer
Statement 1 is correct: For imports, rupee loans. They have to give 75% of their loans to
is exchanged with dollars, so demand of the so-called priority sector, and 50% of the
rupee decreases as compared to dollar. So
loan portfolio should constitute small loans of
rupee destabilises. Thats why reducing non-
up to Rs.25 lakh, even as they will be subject to
essential imports would stabilise imports.
all prudential norms like any other commercial
Statement 2 is correct: While investing bank.
in india, the dollar is converted to rupee,
so demnad for rupee increases and so it 79. (d)
stabilises. Statement 1 is not correct: Gross Enrolment
Statement 3 is not correct: when exports Ratio (GER) is the student enrolment as a
are bought in international Market, rupee is proportion of the corresponding eligible age
needed, so its demand increase. If exports group in a given year. It is nearly 100% in
decreses, rupee destabilises. primary eduction (class 1-5)
Statement 4 is correct: Masala bonds are Statement 2 is not correct: Net Enrolment Ratio
issued in rupee denomination in international is the ratio of children of the official primary
market. So it increases the demand for rupee, school age who are enrolled in primary school
stabilising it. to the total population of the official primary
78. (b) school age.
Statement 1 is correct: Opportunity cost Additional Information
is the the loss of other alternatives when The primary GER indicates how many student,
one alternative is chosen. It will reduce regardless of their age, are enrolled in primary
the opportunity cost for the underbanked school, relative to the population of primary
populace through assisted doorstep banking. school age. The value of the GER can exceed
That means the people would not lose time 100 percent.
and work opportunities while availing

© Copyright:   www.nextias.com

https://t.me/TheHindu_Zone_official
https://t.me/IAS201819 https://t.me/PDF4Exams https://t.me/PDF4Exams

16 | CSE (Prelims) Test Series-2019


80. (b) employed over 40 million workforce in 2013,
Statement 1 is not correct: The GCTF and as per projections is slated to employ over
is an informal, apolitical, multilateral 52 million workforce by 2017, and 67 million
counterterrorism (CT) platform launched in workforce by 2022.
2011. Statement 2 is not correct: NHB RESIDEX,
Statement 2 is correct: There are 30 founding India’s first official housing price index,
Members of the GCTF, including both India was an initiative of the National Housing
and Pakistan. Bank (NHB), undertaken at the behest of the
Government of India, Ministry of Finance
Additional Information:
Recently ninth Ministerial Plenary Meeting of 84. (b)
the Global Counterterrorism Forum (GCTF) Statement (b) is not correct: PM-JAY will
took place. The United States and Morocco cover medical and hospitalization expenses
signed an agreement co-lead the “Initiative for almost all secondary care and most of
on Improving Capabilities for Detecting and tertiary care procedures.
Interdicting Terrorist Travel".
85. (c)
81. (a) Statement 1 is correct: It is an initiative
Department of Economic Affairs releases under National Health Mission
Harmonized List of Infrastructure Sub- Statement 2 is correct: It aims for the
sectors distribution of free generic drugs to
82. (b) government health institutions at all levels
throughout the country.
The Baltic Dry Index, a freight index and a
good proxy for the robustness of trade and Statement 3 is not correct: The initiative
shipping service is released by London based would not only provide support to States
Baltic Exchange. for purchase of drugs but enabling States
to place transparent system of procurement
83. (a) and quality assurance, robust supply
Statement 1 is correct: As per Economic management and logistics that would ensure
Survey: Real estate and construction together, highest level of safety and quallty of drugs
is the second largest employment provider Statement 4 is correct: It will help reducing
in the country next only to agriculture. It the high out of pocket expenditure.

86. (b)
Items
2012-13 2013-14 2014-15 2015-16 2065-17 RE2017-18 BE
(in lakh crore)
Total
26.95 30.00 32.85 33.78 40.60 43.96
Expenditure
Expenditure on
6.58 7.46 7.68 7.90 9.84 10.94
Social Services
(i) Education 3.13 3.48 3.54 3.31 3.95 4.41
(ii) Health 1.26 1.39 1.49 1.52 2.26 2.25
(iii) Others 2.20 2.59 2.65 3.07 3.63 4.27
As percentage to GDP
Total
27.1 26.7 26.4 24.7 26.7 26.4
Expenditure
Expenditure on
6.6 6.6 6.2 5.8 6.5 6.6
Social Services
(i) Education 3.1 3.1 2.8 2.4 2.6 2.7
(ii) Health 1.3 1.2 1.2 1.1 1.5 1.4
(iii) Others 2.2 2.3 2.1 2.2 2.4 2.6

© Copyright:   www.nextias.com

https://t.me/TheHindu_Zone_official
https://t.me/IAS201819 https://t.me/PDF4Exams https://t.me/PDF4Exams

Economy, Revision of Test-3 & Current Affairs_190904 | 17

87. (c) districts of 18 States and four Union Territories.


Statement 1 is correct: Statement 3 is correct: MIDH will work closely
Brazil 0.759 with National Mission on Sustainable Agriculture
(NMSA) to wards development of
China [+] 0.752
Micro-Irrigation for all horticulture crops and
India [+] 0.640
protected cultivation on farmers’ field.
Russia [+] 0.816
South Africa [+] 0.699 92. (d)
Statement 2 is correct: Between 1990 and 2017, Non-tariff barriers to trade (NTBs) or sometimes
India's HDI value increased from 0.427 to 0.640 called "Non-Tariff Measures (NTMs)" are trade
barriers that restrict imports or exports of goods
88. (a) or services through mechanisms other than the
Statement 1 is correct: The government has simple imposition of tariffs.
earmarked Rs. 1,69,323 crore for food subsidy. Foreign exchange restrictions and foreign
Statement 2 is not correct: Fertiliser subsidy exchange controls occupy a special place among
has been hiked to Rs. 70,079.85 crore for 2018- the non-tariff regulatory instruments of foreign
19.Petroleum subsidy has been increased to Rs. economic activity. Domestic subsidies and
24,932.8 crore for 2018-19 industry bailouts acts as a Assistance policy trade
89. (a) barrier. Local or domestic content requirements
are Protectionist policies.
Whole sale Price Index is released by DIPP.
Manufacturing/Service PMI in India is reported 93. (b)
by Markit Economics / Nikkei. Statement 1 is not correct: As per international
CPI (IW) is released by Labour Bureau, Ministry rice research institute (IRRI), China is the leading
of Labour & Employment. producer followed by India. Statement 2 is
Index of Industrial Production is released by correct: India is the second highest producer of
CSO. sugar, after Brazil, in 2017.

90. (a) 94. (b)


Statement 1 is correct: It is caused by a decline in Statement 1 is not correct: Amber box subsidies
demand for production in a particular industry are those subsidies which distort the international
Statement 2 is correct: Structural unemployment trade by making products of a particular country
is often brought about by technological changes cheaper in comparison to same product in
that makes the job skills of many of today's another country.
workers obsolete. Statement 2 is not correct: The type of support
Statement 3 is not correct: In a depressed that fits into the developmental category are
industry, structural unemployment takes place measures of assistance, whether direct or
on account of change in the demand pattern indirect, designed to encourage agricultural and
rural development and that are an integral part
91. (d)
of the development programmes of developing
Statement 1 is correct: Promote holistic growth countries.
of horticulture sector, including bamboo
and coconut through area based regionally Statement 3 is correct: Agricultural input
differentiated strategies, which includes subsidies generally available to low-income or
research, technology promotion, extension, post resource-poor producers in developing country
harvest management, processing and marketing, members, and domestic support to producers
in consonance with comparative advantage of in developing country members to encourage
each State/region and its diverse agro-climatic diversification from growing illicit narcotic
features is one of the mission objectives of the crops.
scheme.
95. (c)
Statement 2 is correct: National Horticulture
Statement 1 is correct: The intention behind
Mission (NHM) is one of the sub schemes
of Mission for Integrated Development of the founding of the World Bank was to provide
Horticulture (MIDH) which is being implemented temporary loans to low-income countries which
by State Horticulture Missions (SHM) in selected were unable to obtain loans commercially. The

© Copyright:   www.nextias.com

https://t.me/TheHindu_Zone_official
https://t.me/IAS201819 https://t.me/PDF4Exams https://t.me/PDF4Exams

18 | CSE (Prelims) Test Series-2019


Bank may also make loans and demand policy of Land Resources; and On Farm Water
reforms from recipients. Management (OFWM) component of National
Statement 2 is correct: The term “World Bank” Mission on Sustainable Agriculture (NMSA) of
refers only to IBRD and IDA. Together, these Department of Agriculture and Cooperation.
two organizations provide low-interest loans, 99. (a)
interest-free credit, and grants to developing Statement 1 is correct: The GDI measures gender
countries. Both IBRD loans and IDA credits gaps in human development achievements
support investment projects and programs by accounting for disparities between women
designed to meet priority economic and social and men in three basic dimensions of human
needs. development—health, knowledge and living
standards using the same component indicators
96. (d)
as in the HDI.
The New Urea Policy-2015 (NUP-2015) has been
Statement 2 is not correct: GDI together with
notified by Department of Fertilizers on 25th
the Gender Empowerment Measure (GEM) were
May, 2015 with the objectives of maximizing
introduced in 1995 in the Human Development
indigenous urea production; promoting energy
Report written by the United Nations
efficiency in urea production; and rationalizing
Development Program.
subsidy burden on the government. I
100. (d)
97. (d)
Statement 1 is correct: The World Trade
Partnerships for the goals: SDG17
Organization (WTO) is the only global
Industry, innovation and infrastructure: SDG 9 international organization dealing with the rules
Life below water: SDG 14 of trade between nations.
Life on land: SDG 15 Statement 2 is correct: All major decisions are
Responsible consumption and production: SDG 12 made by the WTO's member governments: either
by ministers (who usually meet at least every
98. (c) two years) or by their ambassadors or delegates.
PMKSY has been formulated amalgamating Statement 3 is correct: The term (MFN) means
ongoing schemes viz. Accelerated Irrigation the country which is the recipient of this
Benefit Programme (AIBP) of Ministry of treatment must nominally receive equal trade
Water Resources, River Development & advantages as the "most favoured nation" by the
Ganga Rejuvenation; Integrated Watershed country granting such treatment.
Management Programme (IWMP) of Department
nnnn

© Copyright:   www.nextias.com

https://t.me/TheHindu_Zone_official

You might also like